Download as pdf or txt
Download as pdf or txt
You are on page 1of 86

Yan.

NURSING PRACTICE I – COMMUNITY HEALTH NURSING


November 17, 2018

Situation 1- According to the World Health Organization (WHO), there are major causes
of non- communicable disease that pose challenges to the live and health of millions of
people and threaten economic and social development of countries.

1. Which of the following are the four chronic disease referred to by WHO?

1. Cardiovascular Disease
2. Cancer
3. Dementia
4. Arthritis
5. Diabetes Mellitus
6. Chronic Obstructive Lung Disease

a. 1,2,3 and 4
b. 1,2,5 and 6
c. 2,3,4 and 5
d. 2,4,5 and 6

2. The WHO reveals that the NUMBER ONE cause of death from chronic diseases
worldwide, using the 2015 estimate, is _______.

a. Tuberculosis
b. Diabetes Mellitus
c. Ischemic Heart Disease
d. Pneumonia

3. In the Philippines, which is the TOP Killer according to the Department of Health
(2009 data)?

a. Asthma
b. Cancer
c. Tuberculosis
d. Cardiovascular Diseases

4. When studying chronic diseases, the multifactorial etiology of illness is considered.


What does this imply?

a. Single organism that causes the disease, such as cholera, must be studied in more
detail.
b. Focus should be on the factors or combinations and levels of factors contributing to
disease.
c. The rise in infectious and communicable disease must be the main focus.
d. Genetics and molecular structure of disease is paramount.

5. Determinants of health to address the development of cancer in a community include:

1. Proximity of the community to chemical plants that emit poisonous gases.


2. High percentage of tobacco use among the residents.
3. Prevailing diet high in processed food and fat.
4. Availability of the health facilities.
5. Men age of women population.

a. 3,4 and 5
b. 1,2 and 3
c. 1,3 and 4
d. 2,4 and 5

Situation 2: As a nurse, knowledge of the legal aspects in nursing is essential to


safeguard self and clients from legal complications. Clients are becoming increasingly
aware of their health rights, thus you should recognize your professional boundaries and
consequences of nonconformity.

6. Your co-worker got a thank you card from a patient’s family with an accompanying
photo of her with the family. She asked you if it would be alright to post the post on
facebook. Which would be your BEST response?

a. “I think that would be okay, but you should ask permission from our head.”
b. “Yes, as long as you ask the family if that would be okay.”
c. “No, that could lead to a malpractice suit by the patient or family.”
d. “No, posting pictures of patients and families on social media is not acceptable.”

7. Identify which action is acceptable as an exception to a nurse’s obligation regarding


confidentiality.

a. Leaving printouts of lab reports on the desk in the physicians’ lounge


b. Discussing a patient’s care with someone who is not involved in patient care
c. Discussing a patient’s condition in a public place as long as his name is not mentioned
d. Reporting certain disease to public health authority

8. The central question in any charge of malpractice is whether the prevailing standard
of care was met. Which of the following statements are TRUE about standards of nursing
care?

1. Basic prudent nursing care is a standard.


2. Any health care provider can determine standards of care.
3. Standards of care are never changing.
4. Standards are based on the ethical principle of non- maleficence.
5. National standards of nursing practice are standards for all nurses.

a. 1,3,4
b. 2,3,5
c. 1,4,5
d. 2,4,5

9. Select the nursing responsibility which can NEVER be delegated.

a. Rotation schedule
b. Evaluation
c. Accountability
d. Complex Procedure

10. For safe nursing practice, you should observe some of the do’s and do not’s in
performing your professional duty. Which ones are NOT acceptable standard of actions
by the nurse?

1. Keeping current year license to practice


2. Being a witness to a patient’s will
3. Following policies and procedures
4. Protecting patients from injuring themselves
5. Accepting money or gift from patients
6. Giving practical advice contrary to the doctor’s orders *

a. 1,4, 6
b. 1,3,4
c. 2,4,5
d. 5,6

Situation 3: Nurse Betty knows that ethics is important in community health nursing
practice and that ethical decisions are conducted in a universal and standard framework.

11. One step in ethical decision- making is to place an ethical issue or dilemma within a
meaningful context. The rationale for this step is:

a. The nature of ethical issues and dilemmas determines the specific ethical approach
used.
b. People cannot make sound ethical decision if they cannot identify ethical issues and
dilemmas
c. Multiple factors affect the interpretation and justification of the ethical issues and
dilemmas
d. Health professionals cannot avoid choice and action in applying ethics in their work
setting.

12. The historical character in nursing whose work in environmental health, providing
care to destitute people, and disease prevention establishing the nurse as a leader and
community-oriented was _______.

a. Lilian Wald
b. Virginia Henderson
c. Margaret Sanger
d. Florence Nightingale

13. Ethical decision making is the component of ethics that focuses on ____.

a. The result of the decision


b. The process of how decision are made
c. The factors impinging on making the decision
d. Who makes the decision
14. The ethical principle that empowers the client (groups or communities) to make
knowledgeable decision is _.

a. Act in accordance to client’s wishes


b. Act in client’s best interest
c. Offer franks and independent advice
d. Keep the client properly informed

15. When nurse betty cannot meet a need of the client, it is most appropriate for her to:

a. Refer the client to an agency that may be able to give assistance


b. Suggest that the client discuss the need with the physician.
c. Encourage the client to pay someone to meet that need.
d. Inform the client that insurance will not pay for the care.

Situation 4: Nurse Rosas makes home visits to assigned barangays and meets families
of different types.

16. One such family is a married couple who has two biologic children living with them
as well as a child from the wife’s first marriage. What type of family is evident?

a. Homogenous
b. Extended
c. Blended
d. Nuclear

17. Nurse Rosas encounters a married couple who is raising three children. Recently, the
wife’s mother moved in. This family should be assessed as ___.

a. Nuclear
b. Extended
c. Alternative
d. Blended

18. When Nurse Rosas assesses a family, which family task would she consider having
HIGHEST PRIORITY for health family functioning?

a. Reproduction of new family members


b. Physiologic maintenance and safety
c. Allocation of family resources
d. Maintenance of order and authority

19. Which documentation of family assessment indicates a healthy and functional


family?

a. Member provide loving and mutual support


b. Under stress, members turn inward so as not to be burden others.
c. Members believe they can depend in each other.
d. Husband holds dominant power over his wife

20. Which of the following scenario BEST demonstrates a continuing healthy family?
a. A couple renews their marital relationship after their children become adults
b. A couple requires their adolescent children to attend church services three times a
week
c. One parent takes care of children. The other parent earns income and maintains the
home
d. A family has strict boundaries that require members to address problems within the
family.

Situation 5: Interdisciplinary teamwork is an important model for delivering health care


to patients. Integrating services among many health providers leads to better outcomes
delivered to underserved populations and communities with limited access to health
care.

21. Professional group productivity and member satisfaction are best achieved by which
of the following leadership styles?

a. Democratic
b. Authoritarian
c. Laissez-faire
d. Autocratic

22. As a member of a professional work-related group you know that:

a. Functional goals are assigned by management


b. The leader is responsible for overall functioning
c. The members summarize progress and allocate assignments
d. The cooperation of all members is required.

23. The nurse is making a referral for the client. What should be the first consideration
for the nurse to do?

a. Acceptability of the referral to the client


b. Cost of the referral service
c. Barriers to making a referral
d. Client’s eligibility for the service

24. Group with a laissez faire leader are likely to be ___.

a. More structured in format


b. Less productive
c. More productive
d. More satisfying to members

25. You have volunteered to be part of a work group whose purpose is to look at ways to
prevent medication error. You know that:

a. Teamwork enhances the probability of goal achievement


b. Group process is not important in a task
c. The leader assumes responsibility for the overall group functioning
d. The goal should be set by management
Situation 6: Some disasters occur relatively frequent in certain parts of the world. The
Philippines is one country that has experienced both natural disasters and human made
disasters.

26. The public health nurse is aware of the importance of the need to perform roles in
preparing for a disaster. The BEST example of one of these roles would be to:

a. Assist in the prevention of injury


b. Identify disaster risks
c. Coordinate emergency care
d. Inspect cluster of houses

27. The residents and local leaders in Municipality X is mapping out potential disaster
locations in the community. This period of planning and preparation is described as:

a. Recovery stage
b. Pre- disaster stage
c. Impact stage
d. Non- disaster stage

28. The nurse provides counselling for victims of flood. This is an example of _____.

a. Primary Prevention
b. Secondary Prevention
c. Mental health care
d. Tertiary Prevention

29. Should a terrorist attack occur, nurses need to be ready, not only in providing quality
care for the victim but ensuring their own safety as well. During a mass disaster drill
stimulating a terrorist attack, the nurse must triage numerous severely ill persons. The
client who should receive PRIORITY care is:

a. Cyanotic and not breathing


b. Apneic and has an apical rate of 50
c. Having seizures and urine incontinence
d. Gasping for breathing and are conscious

30. The nurse is considering how best to educate the community about the potential
threats to health from terrorism. What are the BEST ways to accomplish this?

1. Hold public forum to educate the community about coping with psychology effects.
2. Raise community awareness about increased immigrants to the population
3. Teach the community about sealing windows and doors in the event of a chemical
attack.
4. Raise community awareness about the signs and symptoms of potential biologic
agents.

a. 2 and 4
b. 1 and 3
c. 2 and 3
d. 1 and 4
Situation 7: Public health nurses like Ms. Shalom should be knowledgeable on the
importance of immunity in the prevention of communicable diseases by building the
capacity of the body to restrict harmful microorganisms or viruses from entering it.

31. Nurse Shalom knows that the type of immunity which is longest acting
is____________

a. Natural immunity
b. Passive immunity
c. Artificial immunity
d. Active immunity

32. She learned that the MOST important function of inflammation and immunity
is___________

a. Preventing any entry of foreign material


b. Providing protection against invading organisms
c. Regulating the process of self- tolerance
d. Destroying bacteria before damage occurs

33. Nurse Shalom administers hepatitis B immunoglobin serum to Helen, which will
provide her with passive immunity. One advantage of passive immunity is that it______

a. Encourages the body to produce antibodies


b. Has effects that last a long time
c. Is highly effective in the treatment of disease
d. Offers immediate protection

34. Nurse Shalom explains to a mother whose ten year child just received a tetanus-
toxoid injection that the toxoid vaccine confers which of the following immunity?

a. Long lasting active immunity


b. Lifelong natural immunity
c. Intermediate artificial immunity
d. Short-acting passive immunity

35. Nurse Shalom knows that one of the following is NOT a vaccine preventable disease,
hence no immunity can be given to a child.

a. Measle
b. Polio
c. Hepatitis B
d. Asthma

Situation 9: As a new public health nurses at a Rural Health Unit, Ms. Ofelia reviews
basic concepts related to her position and job.

41. Ms. Ofelia know that the PRIMARY goal of community health nursing is to ________.

a. Enhance the capacity of individuals, families and communities manage their health
needs
b. Contribute to people’s well-being through sustainable development goals projects
c. Increase the productivity of the people by providing them with health services
d. Support and supplement the efforts of the medical profession in illness preventions

42. Community health nursing (CHN) is a field of nursing practice. Which best explains
this statement?

a. The scope of practice depends on the health needs and problems identified.
b. The services are delivered outside of purely curative institutions.
c. The service are provided along with community health volunteers.
d. The practice is conducted in the geographical location of people.

43. Ms. Ofelia is aware that she will be dealing with population as a client. Which of the
following provides the BEST definition of a population?

a. Cluster of individuals who are at-risk of certain health conditions.


b. Group of people sharing the same geographic environment.
c. Collection of people who are beneficiaries of health services.
d. Collection of individuals who share common characteristics.

44. An example of an aggregate – a group of people with common characteristics or


population that Ms. Ofelia should consider in program planning would consist of ______.

a. Patients confined in a district hospital


b. Christians in the community
c. Young children and adolescents attending school
d. People playing games three times a week in a senior citizen club

45. Which Nursing activity is unique to community/public health nurses like Ms. Ofelia?

a. Care for sick and dying


b. Promote livelihood projects assist poor families
c. Home visits to family clients referred by the health center physician
d. Focus on vulnerable groups in the community

Situation 10 – In public health, quality improvement (QI) techniques are used to


improve performance and service delivery.

46. PDCA is the basic structure for most quality improvement (QI) process. In public
health, PDCA is focused on activities that are responsive to community needs and
improving population health. PDCA stands for:

a. Perform-Document-Correct-Act
b. Perform-Do-Check-Analyze
c. Plan-Document-Check-Analyze
d. Plan-Do-Check-Act

47. There are several approaches and tools in (QI). One of these is the Fishbone
diagram, developed by K. Ishikawa. Which BEST describes the Fishbone diagram? It is
____.
a. A cause-and-effect diagram
b. A wholistic approach in problem identification
c. Used in interview sessions
d. Useful in focusing on problem symptoms

48. As a QI tool, the Gantt chart is useful for planning and scheduling projects. Which of
the following statements is INCORRECT about the GANTT chart? It ______.

a. Provides a graphical illustration of resources


b. Plans the order in which you’ll complete tasks
c. Tracks specific tasks in a project.
d. Helps assess how long a project should take

49. The steps in conducting quality improvement consist of the following: Which is the
correct sequence in the QI process?

1. Identify and select strategies


2. Evaluate QI outcomes
3. Implement solution in small scale
4. State the problem
5. Expand scope of QI throughout the organization

a. 1, 2, 3, 5, 2
b. 2, 1, 5, 3, 4
c. 4, 2, 1, 3, 5
d. 1, 4, 5, 3, 2

50. Client satisfaction survey is an important and commonly used indicator for
measuring the quality in health in health care. In the context of public health, which is
NOT a purpose of satisfaction surveys? To ______.

a. Serve as essential sources of information for identifying gaps


b. Directly address the health problems presented
c. Present the likelihood of continuing client-provider-agency relationships
d. Provide data about health-seeking behaviors

Situation 11 – Promoting mental health is equally important in promoting physical


health. Unfortunately, metal health has been largely neglected.

51. Public Health Nurse Victoria is concerned about the impact of metal health on the
community. She is preparing for a presentation about mental health in the Philippines.
Which of the following information should be included?

a. Mental illness is an increasing problem among the upper class.


b. The incidence of depression is expected to decrease within the next 10 months.
increase
c. Almost one per 100 household has a member with mental disability.
d. Post traumatic stress disorder is the most common of anxiety disorders.

52. The PRIMARY reason that mental illness often results in worsening of physical health
problems is because of the _______.
a. Loss of cognitive function
b. Feelings of inadequacy
c. Inability or lack of motivation for effective self-care
d. Side effects of mental health modification

53. Nurse Victoria shared the statistics that intentional self-harm was found to be the
9th leading cause of death among 20-24 years old Filipino adults. Increasing number of
mental health problems may be related to _______. (Select all that apply)

1. Poverty
2. Age of the parents
3. Family unemployment
4. Changes in family structure
5. Lost of community social support

A. 1, 2, and 5
B. 1, 3, 4, and 5
C. 1, 2, 3 and 4
D. 1, 2 and 3

54. Nurse Victoria noted that there were number of poor older women, living alone, who
are suffering from depression. Her FIRST goal is to ______.

a. Improved quality of life


b. Provide recreation and entertainment
c. Reduce suicide risk
d. Improve level of function

55. In promoting mental health, Nurse Victoria is cognizant of her nursing


responsibilities. These include the following, EXCEPT:

a. Teach parents the importance of community of providing emotional support to their


children.
b. Increase general knowledge of the community on knowledge on mental hygiene.
c. Help the community address factors that promote mental well-being.
d. Refer at once cases of bizarre behaviors to a psychiatrist.

Situation 12 – Public Health Nurse Naomi applies model and concepts in epidemiology
relevant to her nursing practice.

56. Nurse Naomi encourages that parents to have their kinds receive measles
vaccination stating that those who are susceptible may have serious consequences. She
also tells them of the benefits of the immunization. Most parents decide to have the
immunization. Which of the following models is used to guide parent’s decision?

a. Pender’s Health Promotion Model


b. Precede-Proceed Model
c. Reasoned Action Model
d. Health Belief Model
57. In this situation, Nurse Naomi notes that the agent in the epidemiolocal triad that
brings about measles is the ______.

a. Genetic susceptibility
b. School
c. climate
d. virus

58. Nurse Naomi has to be alert about the time interval between the invasion by an
infection agent and the appearance of the first sign or symptom of the disease. This is
referred to as ______.

a. Generation time
b. Incubation period
c. Communicability period
d. Virulence

59. Should an outbreak of a gastrointestinal illness resulting from a food-borne


pathogen happens, Nurse Naomi would likely to think that this is a/an:

a. attack rate
b. secular trend
c. point epidemic
d. event-related cluster

60. Which of the following denotes that resistance of an entire community to an


infectious agent as a result of the immunity of a large proportion of individuals to the
agent?

a. Active immunity
b. Passive immunity
c. Natural immunity
d. Herd immunity

Situation 13 – A public health nurse is giving orientation to three beginning nurses who
are hired by the local government unit under the Nurse Deployment Programs.

61. In the first session, the nurse presented the organization chart of the Rural Health
Unit. Which of the following aspects of organizational structure is illustrated in the chart?

1. Type of work being done


2. Extent of coordination to be done
3. Levels of management
4. Line and staff relationship

A. 2, 3, 4
B. 3 and 4
C. 1, 3, 4
D. 1 and 3
62. The PHN wants to emphasize to the new nurses it is important to observe
professional accountability which is taking responsibility for one’s actions. She explains
that accountability can be done by:

1. Asking assistance from the doctor


2. Performing nursing tasks in a safe manner
3. Reporting and documenting assessment and interventions
4. Evaluating client’s response to nursing care given
5. Evaluating the care when there is complaint by the client

A. 1, 2, 3
B. 1, 3, 5
C. 2, 4, 5
D. 2, 3, 4

63. Accountability also includes a commitment to continuing education to stay current


knowledgeable. According to the Guidelines Implementing Continuing Professional
Education for Nurses (CPE), from which of the following can a nurse earn credit units?

1. Participation in seminars
2. Engagement in a research project
3. As peer reviewer
4. Innovations

A. 2 and 3 only
B. 1, 2, 3, 4
C. 1 and 2 only
D. 1, 3, 4

64. The PRIMARY purpose of requiring continuing education is to?

a. Qualify for work abroad


b. Improve nursing competency after registration
c. Comply with the Board of Nursing requirements
d. Ensure legal practice

65. To better address emerging public health issues, a public health nurse enrolls in a
course in:
a. Ethics
b. Research
c. communication
d. Leadership

Situation 14 – As a public Health nurse, you are aware that many members of the
community are poor and underserved, affected their health status.

66. In the Philippines, poverty remain a challenge the following are some facts on
poverty based on the latest (2015) statistical report. Which statement is NOT included?

a. 28 percent of the country’s 97 million people live below the poverty line
b. Fifteen million Filipinos will rise above poverty in 2020.
c. Farmers, fishermen and children consistently posted the highest poverty incidence.
d. More than 12 million Filipinos are living in extreme poverty.

67. In identifying factors that contribution to poverty, homelessness, and poor health of
a deprived segment of a population, which of the following should you assess?

a. Mental illness and community support


b. Provision of social support and basic services
c. Mobility and age of family members
d. Neighborhood environment and sanitation

68. You have noticed that the population in a poor community has increased including
the rise of pregnant women who are undernourished. You coordinate with the local social
welfare department to provide food that are nutritionally adequate. This is considered:

a. Health promotion
b. Tertiary prevention
c. Primary prevention
d. Secondary prevention

69. In another segment of the community, you realize that there are young adult
members who have poor nutritional habits. You know that these are risk factors for
diabetes and cardiovascular disease. These risk factors can affect their ______.

a. Future potential to change their dietary habits


b. Ability to obtain health services.
c. Ability to network social support systems
d. Health status and employment potential

70. You have learned that people with mental and psychosocial disabilities are a
vulnerable group. Who are restricted in their ability to access essential health and social
care. In your advocacy, you will include the following messages, EXCEPT:

a. Persons with mental and psychosocial disabilities should be confined in an institution


b. Employment and job opportunities must be created for people with mental and
psychosocial disabilities.
c. Mental health should be included in services during and after emergencies and
disasters.
d. Mental health services should be integrated systematically into all health services
including primary level care.

Situation 15 – The Department of Health reported that Filariasis is endemic in 45 out of


78 provinces in the Philippines. You are assigned in the province of Saranggani, one of
the endemic areas. You know that Filariasis carries a social stigma owing to the
disfiguring appearance brought by the disease.

71. You call that Filariasis is caused by parasitic nematode known as ________.

a. Wucheria bancrofti
b. Necator americanus
c. S. haematobium
d. Plasmodium falciparum

72. The disease is transmitted to a person through bites of an infected female mosquito,
known as _____.

a. Aedes aegypti
b. Culex
c. Anopheles
d. Aedes poecillus

73. You receive a 36-year-old man who was brought by his wife to be tested for
Filariasis. The most likely diagnostic test that he will undergo is ______.

a. Immunochromatographic test (ICT)


b. Rumpel Leads Test
c. Urinalysis
d. Stool examination

74. A client in the ACUTE stage of Filariasis will include which of the following clinical
findings?

a. Hydrocele, lymphedema, elephantiasis


b. Lymphadenitis, lymphedema and orchitis
c. Orchitis, hydrocele, elephantiasis
d. Lymphangitis, lymphadenitis, epididymitis

75. Effective methods that the government would likely to pursue to eliminate Filariasis
in the country are all of the following, EXCEPT:

a. Vaccination off all susceptible in risk areas and high-risk populations.


b. Pursue annual mass drug administration using two drugs in all endemic areas for five
consecutive years.
c. Intensify health information and advocacy campaigns in its prevention, control and
elimination.
d. Intensify environmental sanitation such as proper drainage and cleanliness of
surroundings.

Situation 16 – In any setting of nursing practice including community health, records


management such as documentation and record keeping, is important.

76. Documentation is a critical component to the delivery of healthcare. It is a tool which


serves many purposes. Which of the following is NOT included?

a. Create a permanent record for the patient’s future care


b. Provide material for discussion
c. Plan and evaluate a patient’s treatment
d. Ensures continuity of care

77. When recording the home visit, it is important for the public health nurse to _____.

a. Document the visit only when there are significant changes


b. Follow the agency format for recording and documentation
c. Complete the charting every Friday of the week
d. Use phrases in outline form

78. The nurse should document intelligently and clearly. There are ways to help protect
against an allegation of falsifying a medical record. These are the following:

1. Date, time, and sign every entry.


2. Make entries soon after care is given.
3. Written legibly.
4. Be thorough, accurate, and objective.
5. Use only approved abbreviations.

a. 1, 2, 3, 4, and 5
b. 1, 3, and 5
c. 2, 3 and 4
d. 2, 4, and 5

79. Public health nurses must know that all records have a lifecycle and retention
scheduling. Clinical records must be retained in the health care facility for:

a. Anytime with client’s permission


b. Ten years from last date of service
c. Two years from date of client’s last visit
d. Five years from date of client admission

80. Controlled Substances such as drugs dispensed and administered, order and
inventory records must be kept in the health care facility for how many years?

a. Six months
b. One year
c. Five years
d. Three years

Situation 17 – Avelina has recently passed the National Licensure Examination for
Nurses. She intends to apply in the Nurse Deployment Project (NDP) of the Department
of Health.

81. The NDP is open to all nurses who fulfill the following qualifications, EXCEPT:

a. Willing to undergo recruitment and selection process.


b. Possesses an official and validation PRC – Professional Identification Card
c. Must be at least 21 years old at the time of application.
d. Physically and mentally fit as shown in the medical certificate.

82. The NDP has been designed by the Department of Health PRIMARY to ______.

a. Improve local health systems in support of Universal Health Care


b. Augment staff of rural Health units
c. Provide experience to new nurses in terms of work realities
d. Prevent nurses from seeking jobs abroad
83. Under this program, Avelina will be hired on a Contract of Services status. She will
receive a monthly Salary (PhP) of:

a. 13,000
b. 18,000
c. 15,000
d. 22,000

84. Nurse Avelina knows that as NDP nurse, she may be assigned in any of the following
areas:

1. Rural Health Units


2. Barangay Health Stations
3. Disaster-prone areas
4. Level 1 LGU Hospitals
5. DOH Hospitals
6. Birthing homes

a. 2, 4, and 6
b. 1, 3, and 4
c. 2, 3, 5 and 6
d. 1, 2, and 5

85. The NDP is a partnership between the DOH and other government agencies and
organizations. Which of the following institutions is NOT included?

a. Department of Social Welfare and Development


b. Professional Regulatory Board of Nursing
c. Local Government Units
d. Philippine Nurses Association

Situation 18 – Risk factors include genetic and physiological factors, age, physical
environment and lifestyle.

86. The presence of any of these risk factors means that ______.

a. Risk modification will have no effect on disease prevention


b. The chances of getting the disease are increased
c. A person with risk factors can get the disease
d. The disease is guaranteed not to get disease

87. A nurse includes well-being in her definition of health. What will the focus be when
the nurse provides care to a family confronting the imminent death of a family member?

a. Reinforcing the need for the whole family to adapt to the impending death
b. Sharing the nurse’s knowledge on the client’s symptoms of grief
c. Discussing the perceived meaning of life with the family
d. Administering pain medication so that the client can be kept comfortable

88. In the care of families, crisis intervention is an important part of _____.


a. Secondary prevention
b. Tertiary prevention
c. Primary prevention
d. Health promotion

89. The Nurse asks Dante, who is being admitted in a district hospital, with uncontrolled
diabetes mellitus, about his employment status. She knows that _____.

a. A person’s compliance is affected by one’s affected economic status


b. Dante’s employment will cause complication to his diabetes
c. External variables have little effected on compliance
d. Diabetes mellitus will affect his client’s work condition

90. During a home visit, a member of the family who has been using marijuana for three
years tells the nurse that he wants to be “detoxified.” It is important for the nurse to
____.

a. Instruct the patient to change his lifestyle


b. Tell the patient that relapses are not tolerated
c. Identify the patient’s stage of change
d. Realize that the patient is ready for change

Situation 19 – As a public health nurse, you may encounter barriers or challenges in


communication. Thus, it is important that you maintain your professional demeanor.

91. During a support group meeting, Nurse Donato, in a teasing manner, has made
several provocative about your appearance and behavior as a group leader. Select your
MOST appropriate response.

a. “Donato, see after this meeting.”


b. “Donato, what you are saying is inappropriate.”
c. “Donato, you are excused from the group.”
d. “What do you think Donato is trying to tell us?”

92. “The dress code in this facility does not include wearing blue jeans while on duty.”
This statement is an example of using which step in giving constructive criticism? *

a. Listing consequences
b. Expressing empathy
c. Describing the behavior
d. Stating expectations

93. Which of the following is an example of a barrier to successful negotiation?

a. Understanding other’s perspective


b. Becoming emotional
c. Viewing negotiation as collaborative
d. Avoiding phrases that are coercive
94. In one of your meeting you reminded the health center staff that there are quite a
number of members in the community who have not reached primary school. Which of
the following is the BEST communication intervention for a client who is illiterate?

a. Use symbols and images


b. Use touch with speech
c. Personalize speech by using first name
d. Speak loudly and clearly

95. You also emphasized that, “communication must be culturally competent to be


effective”. Which of the following BEST reflects this statement?

a. Listen actively to what is said.


b. Reflect on the meaning of the message.
c. Use simple, direct words.
d. Provide an appropriate environment.

Situation 20 – Ms. Nenita will be involved in an interdisciplinary research to be


conducted by the city Health Office to address health problems of the population
covered. To prepare for this tasks, she reviews her knowledge on the research process
and research design.

96. Which of the following study designs that uses information on current health status,
personal characteristics, and potential risk factors will be appropriate?

a. Case-control
b. Ecological
c. Cohort
d. Cross-sectional

97. The type of epidemiologic study that is used to describe a group of persons enrolled
in a study who share some characteristic of interest and who are followed over a period
of time to observe some health outcome is a(n):

a. Cross-sectional study
b. Experimental study
c. Cohort study
d. case control study

98. The research team will use summary indicators of health as method to depict health
status. What is this approach called?

a. Analytic
b. Holistic
c. Descriptive
d. Evaluative

99. Ms. Nenita is specifically assigned to talk individually with community leaders like
the primary grade teacher and the barangay official in charge of the health committee.
Which data collection method will the nurse use?
a. Key informant interview
b. Community mapping
c. Participant observer
d. Social survey

100. Ms. Nenita suggested to use an analytic approach in their research. Which of the
following statements BEST describes the analytic approach?

a. Focuses on the community’s health problems and issues


b. Utilizes health indicators to define population health
c. Views factors that influence health and by which interventions to improve health are
directed
d. Addresses the social determinants of health to solve community’s problem

NURSING PRACTICE II – CARE OF HEALTHY MOTHER/CHILD


November 17, 2018

Situation 1 – Headnurse Mila wishes to successfully change the way her registered
nurses, nursing assistants, and other nursing staff in providing emotional support to the
ward’s patients.

1. In conducting quality improvement, Nurse Mila should look into which of the following
as her basis, EXCEPT:

A. Patient’s complaint data


B. Patient’s satisfaction survey
C. Incident reports
D. Patient’s chart

2. The FIRST step in the Quality Improvement process is which of the following?

A. Assess progress.
B. Examine the data.
C. Confirm the existence of the problem by gathering data.
D. Set goals.

3. The BEST quality improvement intervention that Nurse Mila can do is which of the
following?

A. Maintain ongoing informal communication with staff.


B. Analyze patients complaint record
C. Interview staff individually and in group
D. Recommend increase in salary of nursing staff.

4. Some of the nursing staff seemed to have a sense of entitlement according to nursing
department leadership. They behaved as though they had unlimited job security
(because of the strength of the nursing unions). Which of the following action is the
BEST?
A. Develop a documentation-and-tracking process for the observations
B. Revise job descriptions and performance for all nursing staff to include the new
general care guideline and highlight emotional support.
C. Continue to work on increasing staff accountability for service behaviors.
D. Develop new evidence-based general care guidelines for nurse-patient interactions.

5. Which of the following requires quality improvement? Select all that apply.

1. Patient’s fall/injury
2. Infection
3. Safety
4. Pressure ulcers

A. 1, 2, & 4
B. 3 only
C. C. 1 & 4
D. D. 1, 2, 3, & 4

Situation 2 - Nurse Crissel was talking about HIV – AIDS to a group of teen-agers. Here
are some questions which were asked by them during the open forum.

6. What are some of the general symptoms the PRIMARY HIV infection? The nurse
enumerated the following. Which among these are CORRECT?

1. Fatigue
2. Headache
3. Fever
4. Sore throat
5. Cough
6. Dyspnea

A. 1, 2, 3, 5, 6
B. 1, 2, 4, 5,
C. 1, 2, 3, 4
D. 2, 3, 4, 6

7. When can AIDS be manifested? The nurse answer was. “It can be as early as
_______.”

A. 1 year or as late as 2 years


B. 2 years or as late as 10 years
C. 1 year
D. 6 months

8. Nurse Crissel also asked the participants if they got to know the transmission of HIV
based from her lecture? Which is Not correct?

A. Accidental blood exposure


B. Kissing
C. Unprotected sex
D. Mother to child transmission
9. On the question as to which of the following are the effects of AIDS on pregnancy,
one teenager cited a wrong answer which was ____.

A. Mild weight loss


B. Prematurity
C. Repeated abortion
D. Infertility

10. HIV transmission from mother to infant occur at post-natal period during _____.

A. Bathing
B. Washing of vagina
C. Bottle feeding
D. Breastfeeding

Situation 3 – Ethics is a field of moral science which deals with the morality of human
acts. Registered nurses must be aware that their actions have professional and ethical
dimensions. They should strive to perform their work to the best interest of all
concerned.

11. Which of the following is NOT aligned to ethics in nursing? A Nurse ______.

A. Has the freedom to do what he likes sans responsibility


B. Is obliged to avoid what is wrong and do what is good
C. Is a person capable of knowing what is right or wrong
D. Should have a sense of accountability for his actions

12. A professional nurse has a duty to know and respect the Patient’s Bill of Rights.
Which of the following is NOT included in the Patient’s Bill of Rights. The right to/for
______.

A. A considered and respectful care.


B. Privacy and confidentiality
C. A complete and current information about his illness
D. Expect continuity of care from discharge to full recovery

13. What bio-ethical principle is violated by a nurse if he provides his patient fraudulent
information about his diagnosis and prognosis?

A. Justice
B. Autonomy
C. Beneficence
D. Veracity

14. Which of the following directly VIOLATES the Patient’s Bill of Rights?

A. Informing patients about the billing policies of the hospital


B. Disclosing the HIV result to members of the patient’s family.
C. Honestly telling the patient about his current condition
D. Immediately referring results of laboratory to the physician.
15. Autonomy is the prerogative of the patient to give consent or refusal of treatment
with the EXCEPTION of which of the following situations?

A. Erroneous belief of a head of a church


B. Negative effect of superstition
C. Near death
D. Peer pressure

Situation 4 – Thelma, a mother of a 6-year-old boy Marco has arrived at school to take
her child home because the school Nurse James has verified that he has an inflamed
throat. Nurse James urges his mother to seek treatment because if the causative agent
is beta-hemolytic streptococcus, he may develop a disorder characterized by inflamed
joints, fever, and the possibility of endocarditis.

16. Which of the following would be the possible disorder?

A. Tetanus
B. Rheumatic fever
C. Scarlet fever
D. Influenza

17. Alice’s mother question whether her other children can catch the same disease.
Which of the following should be the nurse’s response? *

A. Your other children should be taking antibiotics to prevent them from catching the
same disease.
B. It is caused by an autoimmune reaction and is not contagious.
C. You appear concerned that your daughter’s disease is contagious.
D. The fact that you brought Alice to the hospital early enough will decrease the chance
for her siblings getting it.

18. In addition to carditis, which of the following should the nurse assess the child?

A. Oliguria and edema


B. Malabsorption and diarrhea
C. Arthralgia and low grade fever
D. Bronchitis and pneumonia

19. After throat swab for culture and sensitivity, the nurse would expect the physician to
prescribe an appropriate antibiotic. Which of the following is the purpose of this
medication? It is to prevent _______.

A. Recurrence
B. Transmission
C. Inhalation
D. Inflammation

20. If left untreated, such condition can progress to which of the following complication?

A. Kidney failure
B. Left-sided heart failure
C. Right-sided heart failure
D. Angina pectoris

21. When planning a teaching program for a child who has recently been diagnosed with
type 1 diabetes, what will be the nurse’s FIRST concern for the Adel and her parents? To
let them ______.

A. Assess their own feelings about diabetes


B. Learn how to monitor blood glucose level
C. Understand why activities must be limited
D. Learn how to administer insulin injections

22. An evening snack is planned for Adel receiving NPH (Novolin N) insulin
INTERMMEDIATE. This will provide _____.

A. Encouragement for the child to stay on a diet


B. Calories to help the child gain weight
C. Nourishment to counteract late insulin activity
D. Energy for immediate utilization

23. When teaching about insulin and its potential for hypoglycemia, the nurse should
include that its PEAK EFFECT occurs in which number of HOURS?

A. 1 to 2
B. 5 to 10
C. 4 to 12
D. 2 to 4

24. When teaching Adel on dietary management, what should the nurse emphasis
MOST?

A. Food in the form of concentrated glucose should be available all the time.
B. Meals should be preferably prepared and eaten at home
C. Food should be weighed on a gram scale all the time.
D. Meals should be prepared separately from the rest of the family.

25. At 7 AM, the nurse receives the information that Adel has a 6 AM fasting blood
glucose level of 180 mg/dL. What should be her PIORITY nursing action?

A. Inform Adel that a complex carbohydrate such as cheese should be eaten.


B. Encourage Adel to start exercising and to continue for 5 minutes.
C. Tell Adel that the prescribed dose of regular insulin should be administered.
D. Ask Adel to obtain again an immediate glucometer reading.

Situation 6 – To ensure patient safety at all times, personal and professional growth of
nursing is a necessity than a choice. Nurse Florence is a beginning nurse in one of the
hospitals in the suburb. During the Orientation Program, the need to enhance her skills,
knowledge and attitude was emphasized.
26. “Commitment to continual learning and active participant in the development and
growth of the profession are commendable obligation” is contained in which of the
following?

A. Code of Ethics
B. RA 10912
C. RA 7164
D. RA 9173

27. Which law declares that the policy of the State is to promote and upgrade the
practice profession in the country?

A. RA 7164
B. Ra 0173
C. code of Ethics
D. RA 10912

28. Nursing programs that are based on needs assessment and needs analysis and
should be offered free is which of the following?

A. Continuing Development Program


B. In-service Training Program
C. Nursing Education Program
D. Post-Graduate Program

29. In three year’s time, Nurse Florence needs to attain certain number of Continuing
Professional Development units to be able to renew her ______.

A. Philippine Nurses Association membership


B. Professional License
C. Professional Identification Card
D. Professional Registration

30. Nurse Florence was asked by her Head nurse why she requested permission to enroll
in the Graduate Program for the second semester. Her reply should be, EXCEPT:

A. To have an impressive resume to enable her to be competitive


B. Connect with people professionally
C. Invest for the future
D. Pursue her interest in Pediatric Nursing in more depth.

Situation 7 – Nurse Carmi made a study on domestic violence against women in a


certain province in Luzon.

31. Since she wanted to capture the essence and emotion of the victims she choose to
use the qualitative design. Which among the statements below is CORRECT about
qualitative design?

A. It commences study at present but consummates at any future time.


B. It allows estimation of relationship between studied variables.
C. It provides insights into attitudes, beliefs, motives and behaviors of target population.
D. It tests result through numerical data and subjects them to statistical analysis.

32. Nurse Carmi plans to gather 6 participants who are victims of domestic violence
where she will base her questionnaires. What technique in qualitative research is
applicable to this plan?
A. Data triangulation
B. Focus group discussion
C. Interview
D. Case study

33. To have a better analysis and interpretation of finding, Nurse Carmi reviewed and
compared them with other findings of previous researches on the same topic. This is
done MAINLY by doing back to which part of the study? The __________.

A. Statement of the problem


B. Related literature and studies
C. Conceptual framework of the study
D. Theoretical framework of the study

34. Nurse Carmi finally decided to make an in depth study of ONLY ONE SUBJECT of
domestic violence. What design will she use?

A. Causality Design
B. Predictive Correlational Design
C. Descriptive Correlational Design
D. Descriptive Case Study Design

35. In as much as the research may have some risks on the part of the subject who is
21 years old, Nurse Carmi must assure the subject of her anonymity, confidentiality and
respect for her human rights. ONE ABSOLUTE WAY is to get an informed and written
consent from whom?

A. Parents
B. Guardians
C. Husband
D. Subject herself

Situation 8 – Hemophilia is a rare blood disorder that affects the body’s clothing factors.
This may result to incapacitation complications to children affected with this disorder. A
3-year-old child Billy was diagnosed with hemophilia.

36. When counselling parents of a child who has recently been diagnosed with
hemophilia, what must Nurse Thelma KNOW about Billy’s condition whose father is
normal and the mother is the carrier?

A. It is likely that all sons are affected.


B. There is a 50% probability that sons will have the disease.
C. Every daughter is likely to be a carrier.
D. There is a 25% chance a daughter will be a carrier.
37. Billy has slipped on the ice and bumped his knee. Which among the following should
Nurse Thelma prepare to administer, as per doctor’s order? Intravenous infusion of
______?

A. Cryoprecipitate
B. Factor VIII
C. C. Factor X
D. D. Desmopressin (DDAVP)

38. Nurse Thelma is providing home care instructions to the mother of Billy. Which of the
following complications should Nurse Thelma tell the mother, should repeated bleeding
continues?

A. Leukemia
B. Hemarthrosis
C. Ecchymosis
D. Hematoma

39. A nurse analyzes the laboratory results of Billy. The nurse understands that the
MOST likely ABNORMAL finding in Billy is which of the following?

A. Partial thromboplastin time


B. Hemoglobin level
C. Hematocrit level
D. Platelet count

40. The nurse is planning a meal that would provide IRON for a child with bleeding
disorders. Which dinner menu would be the BEST?

A. Chicken nuggets, macaroni, peas, cantaloupe, milk


B. Fish sticks, French fries, banana, cookies, milk
C. Ground beef patty, lima beans, wheat roll, raisins, milk
D. Peanut butter and jelly sandwich, apple slices, milk

Situation 9 – Conchita was diagnosed with gestational diabetes. She is 34 years old and
is on her 26 weeks Age of Gestation.

41. Gestational diabetes is said to exist in pregnancy because of the diabetogenic effect
of what hormone secreted by the placenta?

A. Human placental lactogen


B. Human chorionic gonadotropin
C. Estrogen and progesterone
D. Relaxin

42. Which of the following infection will mothers with diabetes have FREQUENTLY?

A. Moniliasis
B. Herpes zoster
C. psoriasis
D. herpes simplex
43. In 36 weeks, amniocentesis was orders by the doctor for this patient. What is the
purpose of this procedure? To detect/determined ______.

A. An increase level of bilirubin


B. Genetic abnormalities
C. Down syndrome
D. Fetal lung maturity

44. Aside from an endocrinologist, to whom should the nurse also plan to refer the
patient to?

A. General practitioner
B. Neurologist
C. Surgeon
D. Hyperglycemia

45. What is the PRIMARY effect to the newborn of a diabetic mother? *

A. Hypoglycemia
B. Anemia
C. hyperinsulinism
D. hyperglycemia

Situation 10 – Patient Clara visits her obstetrician for a positive pregnancy test. The
nurse in the clinic took her history. Clara reported that she has been pregnant four times
before. She has two children at home, one of whom was born at 32 weeks gestation.
She lost a set of twins at 14 weeks and another baby at 12 weeks.

46. In taking the Obstetrical score, how will Nurse Merlita record Gravida, Para and
(TPAL)?

A. G5, P4, T1, P1, A2, L2


B. G5, P2, T1, P1, A2, L2
C. G3, P2, T1, P2, A2, L1
D. G4, P4, T2, P3, A3, L2

47. When questioned, Clara admitted that she sometimes has several glasses of wine
with dinner. Her alcohol consumption puts her fetus at risk for which condition?

A. Alcohol addiction
B. Anencephaly
C. Learning disability
D. Down syndrome

48. To help determine whether Clara is at risk for a toxoplasmosis infection, what will the
nurse ask the client?

A. “Have you ever had osteomyelitis?”


B. “Have you recently has a rubeola vaccination?”
C. “Do you have any cats at home?”
D. “Do you have any birds at home?”

49. Clara asks Nurse Merlita whether she can take castor oil for her constipation. How
should the nurse respond? “No”. It can ______.”

A. Lead to increased absorption of fat-soluble vitamins


B. Promote sodium retention
C. Produce an adverse effects
D. Initiate premature uterine contractions

50. Clara also admitted to Nurse Merlita that she also use cocaine at least once per day,
skipping meals at times. Which nursing diagnosis is MOST appropriate for her?

A. Activity intolerance related to decreased tissue oxygenation.


B. Risk for infection related to metabolic and vascular abnormalities.
C. Impaired gas exchange related to respiratory effects of substance abuse.
D. Impaired nutrition: less than body requirements related to limited food intake.

Situation 11 – Nursing assessment of individuals across the life span requires a nurse to
be familiar with the physiological, cognitive, and psychosocial changes that occur during
each stage of development.

51. The factors that affect the ability of individuals to cope are______. Select all that
apply.

1. Level of development
2. Coping skills
3. Previous experiences with illness and hospitalization
4. Seriousness of diagnosis

A. 2, 3, and 4
B. 1, 2 and 4
C. 1, 2, 3, and 4
D. 1, 2, and 3

52. Appropriate nursing diagnoses for patients with developmental problems are,
EXCEPT:

A. Delayed growth and development


B. Compromised family Coping
C. Impaired social interaction
D. Altered Sleeping Pattern

53. Patient’s participation in the planning phase of the nursing diagnosis depends on
______. Select all that apply.

1. Development Status
2. Psychological condition
3. Level of intelligence
4. Comprehension of his disease
A. 2 & 3
B. 1 & 2
C. 1, 2, 3, & 4
D. 3 & 4

54. When a 15-year-old female patient is so engrossed and worries as to what her
friends think about her, she is categorized to be in the psychosocial development stage.
Nurse Nene needs to inform the mother that this is a normal behavior. In what stage of
the psychosocial development is the patient in?

A. Identity versus Role Confusion


B. Autonomy versus Doubt and Shame
C. Industry versus Inferiority
D. Initiative versus Guilt

55. The Headnurse of Pediatric Ward asked a staff nurse who is assigned to a 3 month
old infant. At what age will her patient start to drink from a cup? The staff nurse’s
answer should be _____.

A. 12
B. 24
C. 9
D. 5

Situation 12 – Nurse Katrina admitted a laboring primigravida, Yolanda, brought by an


anxious taxi driver.

56. Nurse Katrina calms down the similarly anxious patient by telling her that the
DEFINITIVE sign that labor about BEGIN is _____.

A. Rupture of bag of water


B. Lengthening of cord
C. Initiation of contraction
D. Bulging of perineum

57. The discomfort expected with TRUE LABOR contractions is MOST accurately
described by which of the following? Discomfort ________.

A. Begins in the lower back, abdomen and then radiates over the entire abdomen
B. Begins in the fundus and then radiated downward to the cervix
C. Radiates from the umbilicus to the iliac region then to the lower extremities
D. Centers in the fundus of the uterus during the entire contraction.

58. Nurse Katrina completed the vaginal examination on the patient. She recorded:
75%, 7cm, 0. Which of the following is a CORRECT interpretation of the date?

A. Effacement is 5 cm complete.
B. Fetal presenting part is engaged.
C. Dilatation 75% is completed.
D. Acceleration phase of the first stage of labor is about to begin.
59. Nurse Katrina would further estimate that the above data means the cervix is
_______ of its length.

A. ¾
B. 1/3
C. ¼
D. ½

60. When does the membranes of a patient in labor MOST likely to rupture?

A. During the perineal phase of the second stage of labor as the head of the body
crowns.
B. During the active phase of the first stage of labor, usually at the peak of a
contraction.
C. During the latent phase of the 1st stage of labor between contractions.
D. Just immediately before the labor begins.

Situation 13 – Nurses Jessy and Karyl are assigned in high risk maternity ward. Here are
some cases which they handle for the morning shift.

61. Mrs. Purisima, 38 weeks generation is admitted at the High Risk Ward. She was
admitted due to labor pains with cervix of 6 cm. BP was 160/110mmHg. She is given
MgSo4 per IV. How will Nurse Jessy determine that the MgSo4 therapy has been
effective?

A. Labor pains decreased.


B. There is clonus of the ankle.
C. BP is maintained.
D. Seizures are prevented.

62. What would Nurse Jessy monitor to suspect MgSo4 toxicity on Mrs. Purisima?

A. Tingling in the toes.


B. Rapid pulse rate and hyperventilation.
C. Decreased deep tendon reflexes.
D. Hypertension

63. If Mrs. Purisima begins to exhibit signs of labor after eclamptic seizure, what will
Nurse Jessy anticipate to occur?

A. Severe infection.
B. Abruptio placenta
C. Uterine atony.
D. Placenta accrete.

64. Nurse Karyl is assisting an obstetrician in the care of a pregnant woman in labor with
herpes simplex viral infection. The nurse would anticipate which management of the
condition?

A. Administration of anti-viral drug.


B. Vaginal douche.
C. Hot sitz bath TID.
D. Preparation for Ceasarian section.

65. One of the patients of Karyl has been exhibiting signs and symptoms of Rheumatic
Heart Disease (RHD). These are the following, EXCEPT:

A. Fever
B. Abnormal pulse rhythm
C. Petechia
D. Heart murmurs

Situation 15 – Nurse Vergel is newly assigned in the Pediatric Ward in San Isidro Medical
Center. Since he will be taking care of pediatric patients, there is a need for Nurse Vergel
to be abreast with the legal aspects of pediatric nursing care.

66. Which of the following regulates the practice of the nursing profession?

A. Department of Health
B. Professional Regulation Commission
C. Civil Service Commission
D. Professional Regulation Board of Nursing

67. One of the Vergel’s patients to undergo invasive procedure. Based on the Patient’s
Bill of Rights, sequence the persons from whom Nurse Vergel could obtain consent?

1. Parents
2. Legal guardian
3. Next of kin
4. Physician

A. 1, 2, 4, & 3
B. 1, 4, 2, & 3
C. 1, 2, 3, & 4
D. 1, 3, 2, & 4

68. Based on the Right to Privacy and Confidentiality under the Patient’s Bill of Rights,
the patient has the right to demand the following, but NOT ______ pertaining to his care
as confidential.

A. Information
B. Communication
C. Records
D. financial Status

69. The Right to Information does not include ____.

A. Any change in the plan of care before the change is made


B. Extent to which payment maybe expect from Philhealth
C. Discount for the professional fee of the attending doctor(s)
D. Result of the evaluation of the nature and extent of his/her disease
70. A patient has the right to leave the hospital regardless of his physical condition
provided s/he is informed of which of the following? Select all that apply.

1. Is informed of the medical consequences of his/ her decision.


2. Releases those involved in his/her care from any obligation relative to the
consequences of his decision.
3. His/her decision will not prejudice public health and safety
4. Appropriate arrangement has been made to settle the unpaid bill.

A. 1, & 2
B. 1, 2, & 4
C. 3 & 4
D. 1, 2, 3, & 4

Situation 15 – Liza is a new nurse in the Obstetrics Ward. She remember her Clinical
instruction saying that a nurse must learn how to communicate well with her patients.

71. Liza learned active listening. How will Liza demonstrate this on her patient named
Grace? She Should ________.

1. Listen to what Grace verbalizes and observe how she expresses her feelings
2. Demonstrate interest on what Grace is talking about by avoiding her eyes.
3. Maintain a close body posture such as keeping arms crossed and clenching fist.
4. Tune out other thoughts and refrain from interrupting Grace while she is talking

A. 1 and 2
B. 1 and 4
C. 3 and 4
D. 2 and 3

72. Which of the following should Not be considered by the nurse in interpreting
nonverbal messages of a patient?

A. Culture, educational attainment and position in society.


B. Congruency of nonverbal expressions with spoken words.
C. Facial expression, posture, tone of voice and age.
D. Eye contact with all patients at all times regardless of race.

73. Demonstrating what she learned from school in terms of obtaining feedback, Liza’s
APPROPRIATE response to Grace’s complaint, “My breasts are engorged and it is so
painful, “ should be _______.

A. “Tell me about the pain. Is it tolerable, moderate or severe?”


B. “It is alright all breastfeeding mother feel the same as you”.
C. “That’s a sacrifice that mothers should do for her babies”.
D. “Just continue breastfeeding, it will relieve you from your pain”.

74. Liza also recalled her learning on how to REFOCUS conversations. Thus, when
Grace’s subsequent messages were, “I have this painful engorged breasts and my
mother-in-law wants me to bottlefeed instead of breastfeed”. What will be Liza’s BEST
answer using refocusing technique?
A. “What were you saying about your mother-in-law?”
B. “All mother s-in-law are just as helpful, so it will be better to just accept her”.
C. “I think we were talking about breast engorgement which brought about.”
D. “What did you do about the pain you felt on your breast?”

75. In the course of Liza’s duty in the OB Ward, she also learned that she has to adjust
her style of communication for different types of patients. Thus, Liza should adjust to
which of the following characteristics of Grace? Select all that apply.

1. Level of usual communication


2. Pace of interaction
3. Display of emotions
4. Full development of topic

A. 1, 2, 3, 4
B. 1 and 2
C. 3 and 4
D. 1, 2, 3

Situation 16 – Nurse Kayla is assigned at the postpartum ward. The following questions
refer to postpartum patients.

76. Amy, a multiparous patient, 28 hours after Ceasarian delivery (CS), who is
breastfeeding, complains of severe abdominal cramps. Nurse Kayla explains that these
are caused by which of the following?

A. Flatulence accumulation after CS.


B. Release of Oxytocin during the breastfeeding session.
C. Healing of the abdominal incision after CS.
D. Side effects of the medications Administered after delivery.

77. Three hours postpartum, a primiparous patient’s fundus is firm and midline. On
perineal inspection by Nurse Kayla, she observes a small constant trickle of blood. What
will the nurse suspect for the patient to have?

A. Uterine inversion
B. Perineal lacerations
C. Retained placenta
D. Bladder distention

78. On the first postpartum day, a primiparous patient complains of perineal pain that
was unrelieved by Ibuprofen 400 mg given two hours ago. Nurse Kayla should assess for
which of the following?

A. Vaginal lacerations.
B. Perineal Hematoma.
C. History of drug abuse.
D. Puerperal infection.
79. Twelve hours after vaginal delivery, Nurse Kayla palpated the fundus of a
primiparous patient and finds it to be firm, above the umbilicus and deviated to the
right. What is the Best thing for Nurse Kayla to do for the patient?

A. Contact the physician for an order of methylergonovine.


B. Gently massage the fundus to expel the clots.
C. Encourage patient to ambulate and to void.
D. Document this as a normal finding in the patient’s records.

80. After instructing a primiparous patient about episiotomy care, which of the following
indicates successful teaching?

A. “I will wipe the area from front to back using a blotting motion.”
B. “I will put ice packs on the site 15 minutes once a day for 3 to 4 days.”
C. “Before bedtime, I will do a cold water sitz bath.”
D. “I’ll use sudsy water to clean the episiotomy.”

Situation 17 – Preterm newborns are considered high risk because of their immaturity to
adapt to the extrauterine life. They need special treatment and specialized process of
nursing care.

81. The goal of a neutral thermal environment is to assist the newborns to stabilize its
temperature that does not drop below which of the following?

A. 98.6 degree F
B. 99.5 degree F
C. 97.7 degree F
D. 96.7 degree F

82. Nurses should be alert when caring for a preterm newborn, particularly for which of
the following signs?

A. Hypercalcemia
B. Premature closure of ductus arterious
C. Meconium aspiration syndrome
D. Hypoglycemia

83. The nurse starts giving information about preterm newborns to the parent. She tells
them that preterm newborns exhibits which of the following characteristics/signs? *

A. Allow elbows to be brought past the midline of the chest.


B. Exhibit an abundance of brown fat.
C. Resists when the right heel is brought to the right ear.
D. Assume significant flexed posture.

84. Hypothermia is common in preterm newborns because of their inability to control


heat. Which of the following is an EXCEPTION to the APPROPRIATE nursing intervention
to prevent heat loss?

A. Using mechanical pressure


B. Drying and wrapping the baby
C. Placing the crib beside the wall
D. Doing kangaroo care

85. Which of the following conditions is NOT INCLUDED among the effects of cold stress?

A. Hypoglycemia
B. Metabolic acidosis
C. Increase intracranial pressure
D. Cerebral palsy

Situation 18 – Nurse Melissa has been assigned in the Pediatric Ward for almost a year.
Her Headnurse is on Maternity Leave for the next 6 months. She was appointed by the
Area Nurse Supervisor to be the Charge Nurse.

86. Nurse Melissa recalls that one of the major responsibilities of a Headnurse is record
keeping. These are many reasons for keeping records in health care, but two stand out
above all others. These are to ______.

1. Compile a complete record of the patient’s journey through services


2. Enable continuity of care for the patient both within and between services.
3. Make available for legal purposes
4. Protect nurses from ligitation

A. 1 & 4
B. 2 & 3
C. 3 & 4
D. 1 & 2

87. The following statements demonstrate to Nurse Melissa that record keeping is good,
EXCEPT:

A. Entries are dated and timed as close to the actual time of the events as possible.
B. Relevant information about the condition of the patient/ client at any given time and
the measures you have taken to respond to their needs.
C. Abbreviation is used.
D. A full account of your assessment and the care you have planned and provided.

88. On record keeping, the Area Supervisor reminds Nurse Melissa that for most Nurses
who find themselves involved in legal proceedings, the best defense to win the case is
_____.

A. Honesty
B. Good memory
C. Quality of nursing documentation
D. Knowledge

89. Which statement is incorrect regarding record keeping?

A. Patients now have a legal right to see their own records.


B. Keeping records of patient care and treatment is NOT an integral part of nursing
practice.
C. Any later alteration or addition must be carefully indicated, insuring that the original
entry can still be clearly read.
D. All records must be signed by the person making the entry together with the date and
time of the entry.

90. When the patients asks where to get his/her medical record after discharge, the
correct answer is _______.

A. Office of the Attending Physician


B. Medical Record Section
C. Admission Section
D. Office of the Administrative Officer

Situation 19 – Nurse Myrna, a nurse in Barangay Mayamot Health Center, will conduct
Health education to the residents of the Barangay on Reproductive Health.

91. As a Community Nurse, Nurse Myrna should be able to demonstrate knowledge of


current laws on reproductive health in which she practices. One of the topic topics that
Nurse Myrna will discuss is RA 10354 which is entitled ________.

A. Reproductive Health Act of 2012


B. The Responsible Parenthood and Reproductive Health Act of 2012
C. The Responsible Parenthood and Reproductive Parenthood of 2012
D. RH Bill

92. This Law provides reproductive health care program to all citizens prioritizing, which
of the following?

1. Women
2. The poor
3. Marginalized
4. Those in vulnerable or crisis situations

A. 1, 2, 3 and 4
B. 1, 2 & 3
C. 1 only
D. 1 & 2

93. When conducting health education on Reproduction Health, it is important that Nurse
Myrna examines her values because of which of the following reasons?

1. We all hold personal values that can influence how we respond to our clients.
2. One’s values may change in response to life experiences.
3. Encounters with clients and colleagues may influence your beliefs without your having
much of a chance to reflect on these changes.
4. Others exist at a deeper level, so that we don’t necessarily recognize the influence
they have on our behavior and judgments as health care providers. *

A. 1, 2 and 3
B. 1, 2, 3 and 4
C. 1, 3 and 4
D. 2, 3 & 4

94. Reproductive Health service include the following, EXCEPT:

A. Gender-based Violence
B. Finding the correct partner
C. Management and prevention of maternal and perinatal
D. Malnutrition and anemia

95. Factors that affect reproductive health care, which of the following?

1. Economic circumstances,
2. Education,
3. Living conditions and family environment,
4. Social and gender relationships

A. 1 & 2
B. 1, 2 and 3
C. 1, 2, 3, & 4
D. 2, 3, & 4

Situation 20 – Nurse Miriam Kate is the Charge of the Obstetrics & Gynecology Ward of a
government hospital. Nurse Angelina is one of the staff nurses in that ward. The Ward is
beset by problems such as having inadequate supplies.

96. What is the BEST initiative that Nurse Angelina can do, should there be
non-availability of supply of alcohol used for hand hygiene that is vital in preventing
cross-infection?

A. Requisition of the items from Central Supply before the supply runs out.
B. Inform the Head nurse of its non-availability.
C. Have your own alcohol until hospital supply is available.
D. Request the patients to provide their own alcohol.

97. One major way of preventing cross-infection is frequent handwashing. However,


today, Nurse Angelina just found out there is water interruption. The INITIAL step that
Nurse Angelina ought to do is report the problem to ____.

A. Head nurse
B. Manila Water
C. Chief Nurse
D. Nurse Supervisor

98. Nurse Angelina as a beginning assigned on the Obstetric Ward was asked by Head
nurse Mimi an example of how she will be able to manage her environment. Which of
her responses are CORRECT? Select all that apply.

1. Check availability and expiry date of fire extinguishers in the Obstetric Ward.
2. Compliance of Republic Act 9003 – ecological solid waste management
3. Provision of a separate container for each type of waste for on-site collection properly
marked as “compostable”, “non-recyclable”, “recyclable” or special waste”.
4. Proper isolation of patients known or suspected of harboring infectious diseases.

A. 1, 2 & 4
B. 1, 3 & 4
C. 1, 2, 3 & 4
D. 2, 3 & 4

99. Should Nurse Angelina request for Alcohol-based hand rub to be effective for
application to the hands for reducing the number of viable microorganism on the hands,
the ethyl content should be _____.

A. 60
B. 90
C. 70
D. 95

100. Nurse Angelina discovered that bed mattresses in the Pay Ward slide easily, either
to the left or right side of the bed causing patient fall. What would be Nurse Angelina’s
MOST appropriate way to provide patient’s safety?

A. Use restraint on Patient.


B. Raise upper and preferably lower fall rails at all times.
C. Frequent monitoring of patient.
D. Ensure 24-hour responsible watcher.

NURSING PRACTICE III – CARE OF THE CLIENTS (PART A)


November 17, 2018

Situation 1 – Ms. Limjap is the staff development coordinator of a Tertiary Hospital and
has an In-service training program to a group of newly hired nurses for the month. Due
to the many patients they have in the hospital afflicted with Intestinal and rectal
disorders, a refresher session on these cases became the focus of their discussion.

1. The first question she asked from Nurses Gayle is, how do you differentiate a patient
with Crohn’s Disease from that of ulcerative colitis in terms of the following:

A. Presence of bleeding: severe in Crohn’s Disease while in Ulcerative Colitis bleeding is


mild.
B. Presence of diarrhea: severe in Crohn’s Disease in Ulcerative Colitis it is mild.
C. Affected area: Crohn’s Disease is the descending while in Ulcerative Colitis is the
ascending colon
D. Course of the disease: Crohn’s Disease is prolonged and variable. Ulcerative Colitis
has a remission and exacerbation

2. Nurse Limjap further asked another orientee, Ms. Sharon “In preventing recurrence of
Crohn’s and Ulcerative Colitis, what specific therapeutic regimen is effective to both
condition? The BEST response of nurse Sharon is.

A. Cortecosteroids
B. Anti-cholinergic
C. Antacids
D. Anti-spasmodics

3. As a nurse, Ms. Lyn, what systematic complication do you likely expect when you are
caring for a patient with Crohn’s Disease? The BEST response of Nurse Lyn to Ms. Limjap
is.

A. Megacolon
B. Perforation
C. Small bowel Obstruction
D. Hemorrhage

4. The succeeding questions of Ms. Limjap to the orientees include the following: What
diagnostic test is usually ordered to patients with Ulcerative Colitis in order to distinguish
this condition from other diseases of the colon?

A. Proctosigmoidoscopy
B. Albumin Study
C. Colonoscopy
D. Ultra Sound

5. If you are carrying for patients who are passing out large volume of stools brought
about by Ulcerative Colitis, which of the following nursing diagnoses is the MOST
appropriate to be formulated and incorporated in your nursing care plan?
1. Imbalanced nutrition less than body requirements R/T impaired absorption
2. Rick for Deficient Fluid Volume R/T abnormal fluid loss
3. Risk for Ineffective tissue perfusion R/T low hemoglobin
4. Acute Diarrhea R/T inflammation of the bowel

A. 2 & 3
B. 1, 2 & 3
C. 1, 2, 3 & 4
D. 1 & 2

Situation 2 – Ms. Betsy, 56, year, a former dressmaker, married was brought the medical
counter because of severe abdominal pain radiating to the scapular region. Health
history revealed that she has been fond of eating fatty prepare food. Patient appeared
jaundice with facial grimaces. The physician on duty (POD) examined her and an
impression of Cholelithiasis was given by the POD. Patient was ordered to be admitted
for further work-up and possible surgery.

6. During the admission interview, which of the following questions would the Nurse
APPROPRIATELY asks when a patient is complaining of biliary colic?

A. When do you feel the pain? Is it before you sleep at night?


B. Do you feel the pain and tenderness in the left upper quadrant of your abdomen?
C. Does the pain last for thirty minutes or less?
D. When do you feel the pain, is it after a heavy meal?
7. During the comprehensive assessment of the Nurse, which of the following would the
Nurse likely to find when Ms. Betsay has an obstructive jaundice?

A. Itchiness of the abdomen


B. Clay-colored stool
C. Reddish-colored urine and stool
D. Presence of urobilinogen in the urine

8. The POD ordered the patient for bile duct exploration with open cholecystectomy as
the Xray and diagnostic tests revealed that she has multiple stones obstructing the
biliary tract. After surgery a T-tube was connected as a drain. The nurse on duty (NOD)
found out after twelve hours (12) that there was a remarkable decrease from 250ml to
90 ml in the amount of bile drainage from the T-tube. What is the PRIORITY nursing
action should the NOD do?

A. Report the incident to the physician on duty.


B. Assess the tube for any obstruction
C. Irrigate the tube with the normal saline solution
D. Change the position of the patient.

9. After surgery, Ms. Betsy should be watched for ONE of the MOST common
complications that may arise. The condition is

A. Pleurisy
B. Emphysema
C. Broncho Pneumonia
D. Pneumonia

10. The Nurse on duty is preparing for discharge plan for Ms. Betsay. What dietary
restriction should she emphasize so that recurrence of stone formation will be
prevented?

A. Calamansi Juice, white bread and chicken breast salad.


B. Whole milk, butter, cheese and fried meat.
C. Steamed Fish, broiled vegetable and lean meat
D. Broiled banana, vegetable salad and white meat of chicken.

Situation 3 – Master Gleen is a Senior High School student who has recently been
diagnosed to have bronchial Asthma. Eating seafoods and exposure to animals usually
participate his attack. One day he was rushed to the Emergency Room (ER) because of
asthmatic attack. He was given Theophylline and Cromolyn sodium for his asthma. You
are the nurse-on-duty in the emergency room.

11. When admitting Master Gleen your assessment findings will LIKELY be the following,
EXCEPT:

A. Increased tactile fremitus


B. Resonant to hyper-resonance
C. Wheezing sounds
D. Crackles
12. When administering theophylline to the patient, what PRIORITY drug-induced
adverse reaction should you monitor as a nurse?

A. Constipation
B. Bradycardia
C. Diarrhea
D. Restlessness

13. When evaluating the effects of the drug (Theophylline), what is the positive outcome
do you observe? Decreased in?

A. Breath Sounds
B. Wheezing
C. Body Temperature
D. Allergic Reaction

14. One day, Master Gleen asked his medication Nurse, why he has to take theophylline.
The BEST response of the nurse is it

A. “dilates the bronchial tree and facilitate breathing.”


B. “produces anti-histamine effect and prevent allergic reaction.”
C. “minimize the occurrence of seizure.”
D. “constricts the bronchial tree and decreases inflammation.”

15. What nutritional instruction should you advise Master Gleen when taking
Theophylline? He should AVOID taking?

A. Milk and milk products


B. Fruit and vegetable juices.
C. Soda and Non-carbonated drinks
D. Hot tea or coffee.

Situation 4 – Belen, 42 years old, who is diabetic is brought to the Emergency room
because her husband observed her to be disoriented for the past two days. Her blood
sugar is 300mg/dl. She has fruity breath odor and occasional arrythmia. The physician
suspected that the patient is having Diabetic Ketoacidosis (DKA). You assess the patient.

16. What do you expect the physician to order considering the clinical manifestations of
Ms. Belen?

A. 5% d/nss
B. 0.9 % NSS
C. Ringers Lactate
D. 5% D/W

17. As a nurse, you understand that the Type 2 Diabetes is characterized by the
following, EXCEPT:

A. Onset is acute
B. Insidous in onset
C. Decrease in the number and size of Beta Cells
D. Insulin resistance with Insulin deficiency

18. Ms. Belen started to develop autonomic neuropathy, a common complication of


diabetes mellitus. You are aware that this condition can lead to what condition?

A. Gastro-intestinal and urinary incontinence


B. Development of deep ulcer formation
C. Decreasing vision when ambulating
D. Tiredness and easy fatigability

19. While in the hospital, the physician in coordination with the nurses, dietitian and
other health professionals has to consider total patient care. What particular meal plan
will be carried out by the nurse for Ms. Belen? Food has to be

1. 60 to 70% caloric intake from CHO and monosaturated fats


2. Prepared from lower monosaturated oil like olive oil
3. Prepared from avocado oil and nuts
4. High in CHO, high in saturated fats and low protein

A. 1, 2 & 3
B. 2 & 3
C. 1, 2, 3 & 4
D. 1 & 2

20. Ms. Belen will be discharged in a few days time. What should be include in your care
plan as part of your foot infection prevention?

A. massage feet and legs with mild lotion


B. Use tight stockings when ambulating
C. Trim toe nails with ordinary nail cutter
D. Use heating pad whenever patients feels cold

Situation 5 – Rafael is the Nurse in Charge of the Central Supply Room (CSR) in Hospital
Kundiman. A 100 bed capacity, she knows that she has to distribute supplies to the
different wards in a manner that is equitable and just.

21. What should be her FIRST strategy in order to meet her objective? To collect data on

A. The weekly wastages of supplies


B. Months where consumptions are high
C. The daily consumption of supplies
D. Frequency of events like disasters

22. Time is an important resource in managing supplies. Hence, it is BEST for the nurse
who manages the CSR to requisition supplies and materials _____ days prior to their
consumption. Which is this?

A. 40
B. 30
C. 50
D. 60
23. During rainy days, there are more 0-6 years old pediatric patients who are
hospitalized due to respiratory and gastrointestinal diseases. Hence, there are more who
would need intravenous therapy, what size of cannula must be ordered by the nurse?
Gauge.

A. 24-26
B. 18-22
C. 16-18
D. 22-24

24. Which of the following is the correct principle of stocking intravenous fluids in CSR?

A. A mix of previous and present stocks


B. Present stock first
C. Separation of the present from previous stocks
D. Previous stock first

25. In order to find out oxygen wastes. Nurse Rafaela must advise the ward nurse to
keep track of it on which of the following time frame? At Least every?

A. Shifts
B. Day
C. Weak
D. Hour

26. Safety and quality care are considered priority by the surgical team before, during
and after surgery. The quality outcome indicators for surgery that is ALARMING to the
team includes which of the following?

A. Patients with appropriate specific operative hair removal completed.


B. Prophylactic antibiotic discontinued within 24 hours after surgery.
C. Indwelling catheter removed within 24 hours post-operatively
D. Cardiac surgery patients uncontrolled 6 AM blood glucose post-operatively.

27. One of the Risk Factors for patients going for surgery is nicotine use. If patients are
cigarette smokers, which of the following Nursing actions has to be implemented by the
nurse post-operatively?

1. Encourage breathing, coughing and early ambulation.


2. Offer fluid intake 2500-3000 ml per day, if not contraindicated.
3. Apply nicotine patch to reduce withdrawal symptom, if ordered.
4. Support efforts to quit smoking through smoking cessation program.

A. 2 & 3
B. 1, 2, 3, & 4
C. 1 & 2
D. 1, 2, & 3 28.
28. When a patient is for surgery, preoperative medications are administered to achieve
specific therapeutic outcomes. Which of the following are examples of
ANTICHOLINERGIC drugs which are administered to reduce oral and respiratory
Secretions?

1. Atropine sulfate
2. Glycopyrrolato
3. Scopolamine
4. Ranitidine

A. 1, 2, 3 & 4
B. 1, 2 & 3
C. 2 & 3
D. 1 & 2

29. Anesthesia is used to produce sedation reflex loss and muscle relaxation during a
surgical procedure. When a patient is for surgery of the lower abdomen, perinuem and
lower extremities, the surgeon will likely order what TYPE of anesthesia?

A. General
B. Epidural
C. Conscious
D. Spinal

30. Based on the World Health Organization surgical safety checklist, which of the
following is NOT in the sign-out area of the checklist?

A. Surgeon, anesthesia professional and nurse verbally confirm patient, site and
procedure to be done.
B. Nurse confirms with the team instruments, sponge and needle counts are correct.
C. Whether there are equipment problem to be addressed.
D: Nurse confirms, with the team name of procedure recorded and how specimen is
labeled.

Situation 7 - Ethical decision making in clinical health care is becoming more complex,
this is brought about by the advancement of science and technology, cultural beliefs and
patient's rights.

31. When a patient refuses to undergo insertion of a pacemaker despite the presence of
a third degree AV block and respected by the health team is what bioethical principle?

A. Beneficence
B. Autonomy
C. Nonmaleficence
D. Justice

32. The health care professionals consider the following as ethical dilemma EXCEPT
which ONE of the following. When the _____.

A. answer will produce profound effect on the situation


B. choice is made between equally unfavorable alternatives on basis of risks and benefits
C. situation involves a conflict between two moral imperatives
D. situation cannot be solved by a review of scientific data

33. Nurse Blanca brought her patient to the chapel by wheelchair as promised after
checking the blood pressure of another patient in the ward, is an example of what
ethical principle?

A. Veracity
B. Justice
C. Beneficence
D. Fidelity

34. A newly hired graduate nurse Belen is on the second week of orientation by the
In-service coordinator. Two of the nurses in the Intensive Care Unit (ICU) called "ON
sick" so Ms. Belen was pulled out by the Supervisor to report to the ICU as she could not
get a replacement due to staffing shortage. This ethical dilemma in management is an
example of ______.

A. professional ethics
B. governance ethics
C. organizational ethics
D. clinical ethics

35. Ms. Simon who is assigned in the Emergency room was attending to an older woman
who was having severe abdominal pain with a very unstable blood pressure. The doctor
ordered a very high dose of morphine So4 which Nurse Simon questioned. What ethical
principle is applied in this situation?

A. Veracity
B. Nonmaleficonce
C. Justice
D. Beneficence

Situation 8 - Quality documentation is an essential component of the nurse’s role in the


Health facilities. Nurse Gladys is assigned in the Female medical ward where most of her
patients have cardiac and vesicular disorders.

36. Nurse Gladys is conducting an assessment of Mrs. Carpio who has edema of the
lower extremities due to congestive heart failure. If edema is graded on a scale of 1 (no
visible edema in the leg) to 4+ (Leg very swollen), of what the will Nurse Gladys
document in her chart if her assessment findings of the edeme is slightly swollen?

A. +0
B. +1
C. +2
D. +3

37. What specific term should Nurse Gladys write in her charting when a patient is
suffering from a change in the angle the nail base greater than 180 degrees due to
congenital heart disease?
A. Peripheral neuropathy
B. Peripheral cyanosis
C. Inflammation of the fingers
D. Clubbing of the fingers

38. Mrs. Carpio has an order of Thiaxice diuretics for her edema. The following is the
entry that you expect to be written in her progress report EXCEPT _____.

A. serum electrolyte monitored everyday


B. weight taken before drug is administered
C. intake and output done every shift
D. drug is administered at 8 o'clock in the evening

39. Which of the following is NOT APPROPRIATE description and written in the Nurses
notes when a patient has a pitting edema?

A. Degree of edema
B. Degree of temperature
C. Duration of indentation
D. Depth of edema

40. M. Ruby 20 year old also a patient of Nurse Gladys is suffering from Rheumatic fever
due to a previous streptococcal infection. In monitoring patient's status, the nurse
should document which of the following TYPICAL manifestations EXCEPT:

A. increase body temperature


B. non-pruritic rashes
C. voluntary muscle spasm
D. migratory joint pains

Situation 9 - Mrs. Cruz, 68 years old, 5 2" tall, weighs 80 lbs. came to the of General
hospital Because of diarrhea and eating problems. Her BMI is <18.5 and has not eaten
anything for almost 12 hours.

41. Which of the following is the APPROPRIATE initial question to be asked to the patient
upon admission?

A. “Does stress give you problem with your eating habits?"


B. “Do you recall the food you have taken prior to your admission?"
C. “Do your family members allot enough budget for your nutritional needs?"
D. “What is your usual eating pattern?"

42. Considering the clinical manifestations presented by the patient, how BEST should
the nurse state her nursing diagnosis?

A. Imbalanced nutrition less than body requirements R/T poor absorption of food.
B. Activity intolerance R/T inadequate caloric intake.
C. Risk for infection R/T poor nutritional status
D. Self-care deficit R/T decreased strength.
43. The doctor ordered a nasogastric tube (NGT) inserted before administering Osmolite
tube feeding. Which of the following nursing measures should the nurse do?

A. Assess the presence of bowel sounds.


B. Evaluate if patient's residual is <50 ml
C. Warm the Osmolite to 98.8 F.
D. Flush the NGT with 10 ml of water.

44. Which of the following results will describe a normal physical finding when
performing gastro-intestinal assessment?

A. Eversion of umbilicus upon inspection.


B. Tympany on percussion of the abdomen.
C. Gurgling abdominal sounds upon auscultation
D. Liver edge of 1-2cm below the right costal margin on palpation.

63. In the process of doing performance appraisal of nursing staff, Nurse Karen is guided
by the basic goals of comprehensive appraisal system. Which of the following is included
in this process?

1. Determine the capability required for the job


2. Match abilities of the nurse with the job requirements
3. Improve nurse's capability
4. Enhance nurse’s motivation

A. 1, 2, and 3
B. 3 and 4
C. 1 and 2
D. 1, 2, 3 and 4

64. In conducting performance appraisal, Nurse Karen as a manager, is aware that bias
can happen when assessing her staff with just one method. To minimize subjectivity
multiple methods can be employed. Which of the following methods is MOST applicable
for a staff position level?

A. peer review
B. Management by objectives
C. 360 degree evaluation
D. self-evaluation

65. Nurse Karen has a staff nurse who has been with her unit for almost two years.
Recently, she has observed that her work performance is getting substandard, such that
she misses deadlines and is becoming uncooperative. What is BEST approach to be done
to this problem?

A. Mentoring
B. Scolding
C. Counseling
D. Coaching
Situation 14 - A domain in the development of the competency of the practicing nurse is
ones' Legal responsibility.

66. The graduates from the colleges of Nursing are being prepared to take the Nursing
Licensure Examination (NLE) with the main goal to:

A. provide opportunity for overseas employment


B. determine the standards of nursing practice
C. protect the public from incompetent nurse practitioner
D. limit the practice of the profession

67. Ms. Libby is a professional nurse who was approached by her cousin to be in a
Television show where she has to wear the nurses uniform and advertise a commercially
prepared beauty product for use in their work setting. Her mother is presently having
dialysis so the accepted the offer. Ms. Libby is violating the _______.

A. nursing law
B. nursing code of ethics
C. nursing jurisprudence
D. consumers fraud law

68. Mr. Larson is scheduled for Coronary Artery-by-pass graft (CABG) surgery. He has to
sign an informed consent prior to operation. The validity of this consent is how long?

A. 36 hours after surgery


B. 24 hours after surgery
C. after the surgical procedure
D. 12 hours after surgery

69. A patient was admitted in a medical center by his physician for an Exploratory
laparotomy. After discharge for two days, he was complaining of chills and fever. The
abdominal incision was passing out yellowish purulent discharge. He had an abdominal
Xray and was was found out to have 3 pcs of gauze inside the abdomen. A libel suit was
filed by the family. Who is liable in this case?

1. Surgeon
2. Circulating nurse
3 Anesthesiologist
4. Hospital

A. 1, 2, 3, & 4
B. 1, 2, & 3
C. 1 & 2
D. 2 & 3

70. Nurse Rose is assigned to a patient in ICU pronouned who is dying due to cardiac
arrest. After two hours, the partient was pronouned dead. Which of following health
status should guide the nurse is the patient is declared dead?

A. All body systems are no longer functioning


NURSING PRACTICE IV – CARE OF CLIENTS (PART B)
November 18, 2018

Situation: A landslide had occurred in x province due to persistent and heavy rainfall.
According to information received by the Emergency Department Staff, there were about
150 people injured.

1. The nurse prepares the department to receive the injured. This situation would be
BEST classified as a/an:

a. Unnatural Calamity
b. Natural Calamity
c. Mass Casualty Incident
d. Accidental Disaster

2. The first responders to the scene of disaster would MOST likely use which of the
following tools to separate victims for easy recognition of those in need of immediate
care.

a. Number Codes
b. Triage Tag
c. Category Tags
d. SALT Triage

3. The ED nurse plans the coding for disaster victims. Which colors are MOST associated
with triage?

1. Green
2. Yellow
3. Black
4. Blue
5. White

a. 2, 3, 4, 5
b. 1, 2, 3, 4, 5
c. 1, 2, 3, 4
d. 1, 2, 3

4. The ED nurse receives a victim with severe life- threatening injuries and most likely
will not survive. The victim would be triaged as:

a. Yellow
b. Red
c. Black
d. White

5. One of the victims, a female was brought to the ED crying hysterically and looking
frightened. She exclaims she was inside her house when the landslide occurred and just
barely escaped. What is the BEST action by the nurse? *
a. Refer her to the proper authorities for counselling.
b. Triage her and give appropriate treatment.
c. Call a physician to talk to her.
d. Advise her to go home with a relative.

Situation: A nine year old male child is hospitalized for burns on the right arm, right leg
and abdomen. The nurse documents the treatment performed on the child.

6. The nurse determines the extent of burns using the rule of nines. Which of the
following assessment findings should the nurse document?

a. 18% of the child’s body surface is burned.


b. 45% of the child’s body surface is burned.
c. 50% of the child’s body surface is burned.
d. 25% of the child’s body surface is burned.

7. The nurse writes a nursing diagnosis for the child which is the basis of care for the
first 24 hours of admission. The MOST appropriate nursing diagnosis would be _______:

a. Fear and Anxiety


b. Disturned Body Image
c. Risk for Infection
d. Impaired Mobility

8. The physician writes an order for the client. Infuse D5 water 500 cc to run for 8
hours. The IV micro set delivers 60 drops per ml. How many drops should the nurse
regulate the flow and record it in the client’s chart?

a. 50 drops per minute


b. 62 drops per minute
c. 35 drops per minute
d. 30 drops per minute

9. The nurse commits an error in documenting the care of the burnt child. She consults
the charge nurse to find out if the hospital has an established policy on correcting
documentation errors. Which of the following is an accepted form for correcting errors?

a. Enclose in parenthesis the erroneous statements, draw a line across the statement,
and make the correct entry above the 1 drawn.
b. Correct the error by applying correction fluid or tape and write the correct entry over
it.
c. Cross through the erroneous word or statement with a double line, affix your initials,
write the pharse “mistaken entry” then write the correct information.
d. Use the slide rule method. Cross through the erroneous word or statement with a
single line, affix your initials, write the date and time the correction was made, the write
the correct information.

10. The nurse is aware that documentation requires the following EXCEPT:
Documentation should:
a. Be systematic and organized
b. Comply with policy standards of the health care facility.
c. Present exact and correct details pertinent to the event.
d. Include reactions and interpretations of the nurse on the event.

Situation: The assistant chief nurse of X hospital attends a seminar on quality and
performance improvement. The seminar is to increase awareness on how performance
improvement affects client care and the health care organization.

11. Which of the following principles of quality improvement is MOST appropriate for
patient care?

a. The priority is to benefit clients and all other internal and external customers.
b. Improvement of the quality of service is a continuous process.
c. Improvement opportunities are developed by focusing on the work process.
d. Quality is achieved through the participation of everyone in the organization.

12. The assistant chief nurse understands that the implication of quality improvement
for client care can be measured by the overall value of care. Outcomes can be measure
by which of the following?

a. Client’s recommendation
b. Clinical out come
c. Cost of care
d. Client satisfaction

13. The assistant chief nurse is oriented on the various improvement strategies at the
organization level. One of these strategies is benchmarking. Which of the following
describes the bench marking process?

a. Comparing data in the literature with the data collected per client.
b. Reviewing own unit’s data for opportunities.
c. Collecting data of the individual client.
d. Comparing ow data with that of other organization to identify opportunities.

14. The assistant chief nurse understands that a sentinel event review is one strategy to
improve the health care delivery system. The FIRST action to be initiated following a
sentinel event is for the assistant chief nurse to ____.

a. Conduct an immediate investigation


b. Conduct a root cause analysis
c. Recommend corrective action of personnel involved
d. Recommend what action personnel involved should avoid

15. Quality improvement can be achieved through the participation of everybody in the
health care organization at all levels. Which of the following is an example of nurses’
participation as an individual?

a. Work with others in the unit to change the way client’s report is given to be more time
efficient.
b. Suggest the process for notifying the pharmacy department about a missing
medication.
c. Participate in a term to find a solution.
d. Change the activities of her day to spend more time with a client’s family.
Situation: Nurse Edna, a charge nurse of an oncology unit attends a seminar on
evidence based nursing practice.

16. The conference speaker explained the difference between evidence based practice
and evidence based nursing practice. Which of the following is NOT TRUE regarding
evidence based nursing practice? Evidence- based nursing practice ____.

a. Has a medical focus


b. Uses a therory derived and research based information in making decision about
nursing care delivery.
c. Considers the patient’s need and preferences based on nursing theory and research
d. Is a strategry used to improve patient care outcome.

17. The speaker encouraged the nurses to participate in the use of evidence- based
nursing practice. The speaker said that nurses must ____.

1. Participate in the development, use, or evaluation of practice guidelines.


2. Read and analyze outcome of research studies
3. Involve themselves in everyday patient care and nursing practice.
4. Know why they are doing what they are doing

a. 2, 3 and 4
b. 1 and 2
c. 1 ,2, 3 and 4
d. 1, 3 and 4

18. Nurse Edna is aware of the importance of patient outcomes as a measure of quality
care. Which of the following statements is TRUE regarding patient-focused outcome
indicators? To _______.

1. Achieve safe, quality, cost effective care for patients in daily practice.
2. Realize that individual nursing practice styles directly affect the rates at which
patients recover.
3. Prevent development of unnecessary complications and injury.
4. Determine satisfaction rate from patients or family care received from nursing staff.

a. 1, 2 ,3 and 4
b. 1 and 3
c. 2 and 4
d. 1, 3 and 4
19. A model for using evidence- based practice was presented. The model has the
following elements: Plan, Do, Study and Act (PSDA). If nurse edna wants to utilize this
model to improve ward management, what questions will she ask?

1. What are we trying to accomplish?


2. How well do we know that a change is an improvement?
3. Will the patient be satisfied with the improvement?
4. What change/changes can we make that will result in an improvement?

a. 2 and 4
b. 3 and 4
c. 1 and 3
d. 1,2,3 and 4

20. Nurse Edna is aware that patient care improvement must be based on which of the
following?

a. Total cost of health care


b. Building and apply knowledge
c. Hospital policy on how to staff a nursing unit.
d. Accreditation standards

Situation: The nurse cares for a client with cancer who had lung surgery. The nurse
educates the client on breathing exercise and ambulation.

21. The nurse teaches the client how to deep breath effectively after a lobectomy. The
nurse instructs the client to:

a. Contract the abdominal muscles, take a deep breath through the mouth and exhale
slowly as one trying to blow out a candle.
b. Relax the abdominal muscle, take a slow deep breath through the nose, and hold it
for 3 to 5 seconds.
c. Relax the abdominal muscle, take deep breath through the mouth and exhale slowly
for 15 seconds
d. Contract the abdominal muscles, take a slow deep breath through the nose, and hold
it for 3 to 5 seconds.

22. The client asks the nurse how much of his lungs are removed. The nurse responds
based on information that in lobectomy, a lobe is removed. In a wedge resection, which
of the following is removed?

a. A small, localized are near the surface of the lung


b. One entire lung
c. Two lobes of the lung
d. A segment of the lung, including a bronchiole and its alveoli.

23. The client asks the nurse, what will fill the space where the lobe was? The correct
response would be: The _____.

a. Lung space will be filled up with serous fluid.


b. Surgeon filled the space with gel.
c. Space stays empty
d. Remaining lobe or lobes over expand to fill the space.

24. On the second post-operative day, the nurse auscultates the lungs and determines
scattered crackles bilaterally. Which of the following interventions would be MOST
appropriate for the nurse to perform?

a. Encourage deep breathing and ambulation as soon as the client is able.


b. Encourage coughing and check the water seal system
c. Reduce the frequency of pain medications and increase the suction in the water seal
bottle.
d. Perform endotracheal suctioning every shift.

25. The nurse teaches the client to perform which of the following exercises to prevent
shoulder ankylosis?

a. Raise and lower the head


b. Turn from side to side
c. Raise the arm on the affected side over the head
d. Flex and extend the elbow on the affected side

Situation: Nurse Elma speciaized in emergency nursing. She collaborates with ED


physician, triage team and other members of the health team provide care to clients of
all age- groups with various illnesses or injuries coming to the emergency department.

26. When clients are wheeled into the ED, Nurse elma applies a triage priority rating
system. When a client must be treated immediately otherwise the client’s life, limb or
vision are threatened, the category is ____:

a. Low urgent
b. Urgent
c. Emergent
d. Non Urgent

27. A client with spinal cord injury due to trauma is brought to the ED by EMS personnel.
Nurse Elma assesses the client. Which of the following will the nurse suspect the client
to manifest?

a. Tachycardia
b. Diaphoresis
c. Pain
d. Temporary Loss of relax function

28. The neurosurgeon performs a neurological assessment on the client. Based on the
Glasgow Come Scale, the client has a total score of 8. Nurse Elma recognizes this score
to indicate which type of injury?

a. Severe injury
b. Minor Injury
c. Moderate injury
d. No injury

29. Nurse Elma writes a nursing diagnosis. Which of the following diagnoses is a
PRIOPRITY nursing diagnosis for a client with spinal cord injury?

a. Risk for infection


b. Dysreflexia
c. Ineffective airway clearance
d. Ineffective breathing pattern
30. The neurosurgeon determines the client’s spinal cord injury is at the level of T5.
Nurse Elma is alerted when the client complains of severe headache, is diaphoretic, a
head and neck appear to be flushed. She takes the vital signs. Plus rate is 47 beats per
minute and blood pressure is 220/114 mmhg. Based on the assessment data, nurse
Elma concludes that the client need IMMEDIATE treatment for _____:

a. Spinal shock
b. Autonomic Dysreflexia
c. Pulmonary Embolism
d. Malignant Hypertension

Situation: Nurse Tina admits a 25 year old female for severe angioedema involving the
face, hands and feet. The patient further complains of burning and stinging of the
lesions.

31. The nurse interviews the client. Which of the factors would be a MOST significant
risk factor for allergies?

a. A recent upper respiratory infection


b. Family history of allergic rections
c. Living in a third world country
d. Exposure to fungal infection

32. Based on assessment findings, nurse tina writes a nursing diagnosis is:

a. Risk for Injury


b. Altered Comfort
c. Risk for Infection
d. Impaired skin integrity

33. The patient undergoes allergen testing using the cutaneous scratch method. To
prevent anaphylaxis, nurse Tina should initially monitor the patient’s ____.

a. Bilateral lung sounds


b. Arm at the site of the skin testing
c. Pupil’s size and reaction to ligh
d. Blood pressure and pulse

34. Nurse Tina knows that allergic reactions usually occur within minutes after injection
of an allergen. The nurse should ____.

a. Administer high flow oxygen


b. Document the patient’s allergy history
c. Observe the patient for about 20 minutes after injection
d. Monitor the patient’s blood pressure and pulse

35. The physician orders Epinephrine 1:10,000 0.5ml IV. After administering the drug,
the NEXT action of nurse tina is to ___.

a. Start oxygen at 100% using a non-breather mask.


b. Prepare an infusion of dopamine (Intropin)
c. Administer diphehydramine (Benadryl) IV.
d. Give a dose of cimetidine (Tagamet).

Situation: A 51 year old male is admitted for complaints of rectal bleeding, abdominal
pain, weight loss and change in bowel habits. Nurse Lawrence is aware that these are
manifestations of colon cancer. The physician prescribes radiation therapy and
chemotherapy.

36. Nurse Lawrence knows that radiation therapy is used to treat colon cancer before
surgery. Which of the following is the effect of radiation therapy? It ____.

a. Help heal the bowl after surgery


b. Eliminates malignant cells
c. Cures the cancer
d. Reduces the size of the tumor

37. The patient undergoes radiation therapy. Nurse Lawrence noted that the patient’s
white blood cell (WBC) count is severely depressed. The PRIORITY nursing intervention
would be to ____:

a. Place the patient in a private room and maintain strict aseptic technique for all
procedures.
b. Instruct the patient to avoid shaving with a sharp razor
c. Encourage visitors to visit the patient regularly to reduce the feelings of isolation the
patient may feel.
d. Encourage the patient to include fresh fruits and green leafy vegetables in his diet.

38. Nurse Lawrence is aware that chemotherapy should only be administered by nurses
who have taken special courses in administering chemotherapy and who are highly
skilled. Before the nurse gives the prescribed dose of chemotherapeutic agent, the nurse
should do which of the following:

a. Verify the dose, drug and schedule with another trained nurse.
b. Collect an extra syringe and needle in case of contamination.
c. Explain the expected side effects of the drug to the patient
d. Cover the patient with a water resistant shield

39. Nurse Lawrence assesses the patient receiving chemotherapy. Select the signs and
symptoms that would require further evaluation.

a. Patient complains of fatigue


b. Hair loss on scalp
c. Large areas of ecchymosis in various sites on the body.
d. Dry mucous membrane

40. Nurse Lawrence writes a care plan for the patient receiving chemotherapy. Select
what should be included in the care plan.

1. Nursing Diagnosis
2. Medical Diagnosis
3. Outcome
4. Interventions
5. Patient Education
6. Evaluation

a. 1, 2, 3, 4, 5
b. 1, 2, 3, 4, 6
c. 1, 3, 4, 5, 6
d. 2, 3, 4, 5, 6

Situation: Nurse Elma assists in the care of a 25 year old male who is admitted in the
emergency department for burns in the chest, abdomen, right arm and right leg.

41. The physician orders total parental nutrition (TPN) for the burn patient. Which of the
following statements is TRUE in this case? TPN is needed to _______.

a. Provide supplemental vitamins and minerals


b. Correct water and electrolyte imbalances
c. Ensure adequate caloric and protein intake
d. Allow the gastrointestinal tract to rest

42. Nurse Elma is aware that fluid shifts occur during the emergent phase of a burn
injury. This shifting is due to fluid moving from what space? From ________.

a. Intracellular to extracellular space


b. Extracellular to intracellular space
c. Vascular to intracellular space
d. Interstitial to vascular space

43. Nurse Elma understands that fluid shift results from an increase of which of the
following?

a. Total volume of intravascular plasma


b. Total volume of circulating whole blood
c. Permeability of the kidney tubules
d. Permeability of capillary walls

44. Which of the following fluid and electrolyte imbalance would Nurse Elma anticipate
that the patient would be particularly susceptible to in the emergent phase of burn care?

a. Hyperkalemia
b. Metabolic Alkalosis
c. Hemodilution
d. Hypernatremia

45. The patient is ordered to receive fluid resuscitation therapy. Nurse Elma adjusts the
infusion rate by evaluating the patient’s:

a. Hourly body temperature


b. Hourly urine output
c. Hourly urine specific gravity
d. Daily body weight
Situation: Nurse Flora admits a 30 year old female with tentative diagnosis of
hypercalcemia.

46. Nurse Flora recognizes the signs and symptoms of hypercalcemia. Which of the
following signs is an indication of the diagnosis hypercalcemia?

a. Positive trousseau’s sign


b. Hyperactive bowel sounds
c. Hypertonicity of the muscles g form answer
d. Twitching
e. No Answer

47. Nurse Flora recognizes the signs and symptoms of hypercalcemia. Which of the
following signs is an indication of the diagnosis hypercalcemia?

a. Muscle strength
b. Blood Pressure
c. Weight
d. Edema

48. The patient informs the nurse that she is taking Thiazide diuretics. Nurse Flora
knows that Thiazide diuretics drug are one of the most common causes of
hypercalcemia. Which of the following signs should the nurse observe for?

a. Increased peristalsis
b. Neurologic Depression
c. Neuromuscular Irritability
d. Decreased urine output
49. The patient goes into hypercalcemic crisis. Family members are anxious and worried.
One relative expresses to Nurse Flora “We don’t know what to do if she dies”. What is
the BEST response of the nurse?

a. “Do not worry. We always see this kind of crisis and we can treat it.”
b. “I understand your concern, but I have to talk care of the patient first”.
c. “Yes, it is serious but I can come back, talk to you and answer your questions.”
d. “Has your loved one been eating processed food and drinking alcoholoic beverages
lately?”

50. Fortunately, the patient recovers from the hypercalcemic crisis. The patient is ready
for discharge. Which of the following activities should be included in the discharge plan?
Instruct the patient to ____.

a. Take in anti-diarrheal medications as prescribed by the physician


b. Encourage foods that increase urine acidity
c. Decrease sodium and calcium intake
d. Restrict fluid intake to less than 1 liter a day

Situation 11 – Mr. M.E., 37 years old, was accompanied by his wife and teen-age
daughter to the out-patient department for complaints of fever, fatigue, malaise and
painful swollen joints. The physician ordered that Mr. M.E. be admitted to the hospital for
observation and treatment. You are the admitting nurse in the OPD. You found out
during the interview that Mr. M.E. does not have a regular job. His wife works as a
laundry woman.

51. Before admitting the client, you should FIRST make sure that:

a. The consent for admission is signed by the client.


b. The consent for admission is signed by the wife and witnessed by the daughter.
c. The client can pay his hospital bills.
d. The consent form is signed by the social worker.

52. Mr. M.E id brought to the medical ward. The next day, he wants to know about his
illness. The nurse on duty replied, “You don’t need to know your diagnosis”. Which of
following rights of the patient is violated? Right to ______.

a. Obtain from his physician complete current information concerning his diagnosis,
treatment and prognosis.
b. Receive from his physician information necessary to give informed consent.
c. Expect reasonable continuity of care.
d. Considerate and respectful care, irrespective of one’s socio- economic status.

53. After five days of hospitalization, the physician said Mr. M.E can be discharged. He
ordered medications to be taken at home. The client is still weak and symptomatic.
Which of the following rights could be violated in this case? Right to ____.

a. Know hospital rules and regulation


b. Privacy
c. Refuse treatment
d. Continuity of care

54. Because Mr. M.E cannot pay for his medical bills, he is referred to the social worker.
Which of the following rights is applicable on this case? The right to ___.

a. Considerate and respectful care irrespective of his socio-economics status.


b. Expect reasonable continuity of care
c. Examine and receive an explanation of his medical bills regardless of the source of
payment
d. Know what hospital rules and regulation apply to his conduct as a client.

55. The nurse discusses and shares the medical records of Mr. M.E to a group of visiting
members of a medical mission team. Which of the following rights could be violated? The
right to ___:

a. Expect that all communications and records pertaining to his care should be treated
as confidential.
b. Obtain information regarding any relationship of the hospital to another health care
and educational institution in so far as his care is concerned.
c. Informed consent
d. Privacy
Situation 12 - Ms. Jenny is a charge nurse of an oncology unit. She prepares a unit plan
to improve the delivery of patient care and maximizing all human and material resources
of the unit.

56. Ms. Jenny implements a change in the nursing delivery system from functional to
team nursing. This type of nursing model of practice is:

a. Nursing personnel are led by a registered nurse in providing care to a group of


patients.
b. A task approach method used to provide care to patients.
c. Managed care concepts and tools are used in providing patient care.
d. A single registered nurse is responsible for providing care to a group of patients.

57. In implementing the change, Ms. Jenny has the consensus of the staff and other
personnel involved in the care of patients. The main purpose of achieving a consensus
when making a decision within a group is to ___.

a. Demonstrate that staff member are flexible


b. Facilitate cooperative effort toward goal achievement
c. Explore possible alternate solutions
d. Ensure the use of effective autocratic decision making

58. Ms. Jenny is aware that there are nursing and other personnel who may be resistant
to the change and are not taking an active part in facilitating the process of change. To
overcome resistance to change, the MOST important action by Ms. Jenny would be to
___.

a. Emphasize the positive consequences of the change


b. Identify the reason or reasons for the resistance
c. State clearly and concisely the purpose of the change
d. Modify the objectives to appeal to more people holding key positions

59. To ensure efficiency when organizing daily workload, Ms. Jenny reminds her staff
that they should do which of the following:

a. Organize care around legally required activities.


b. Plan activities to promote nursing convenience
c. Perform routine bed baths between 8:00 to 10:00 in the morning.
d. Provide care to a patient in isolation first.

60. Which of the following is the MOST effective resource in helping a staff nurse identify
a solution to a clinical problem?

a. Other staff nurses in the unit


b. Nursing procedure manual
c. Organizational chart of the hospital
d. Nurse manager of the unit

Situation 13 - A 55 year old male is admitted for complaints of joint pains, weakness,
muscle spasm and stiffness when waking up in the morning. He was diagnosed with
Rheumatoid Arthritis (RA)
61. Nurse Ray is aware that RA is a debilitating chronic auto-immune disease. The
patient asks the nurse why his joints are becoming increasingly painful. Which of the
following statements is TRUE about RA? Rheumatoid Arthritis ____.

a. Is usually caused by the aging process


b. Results from degenerative joint damages
c. Begins with inflammation of joints synovial tissue
d. Affects only the weight bearing joints of the body

62. The patient further complains that he gets tired easily when helping his wife in
household chores. He further complains that whenever he walks, his knee hurts. Based
on this information, Nurse Ray writes a nursing diagnosis. Which of the following nursing
diagnoses is MOST appropriate?

a. Activity intolerance related to fatigue and pain


b. Body image disturbance related to fatigue and pain
c. Ineffective individual coping related to chronic pain
d. Self- care deficit related to increasing joint pains

63. Nurse Ray develops a care plan for positioning the patient. Nurse Ray instructs the
patient to assume a position that would ___:

a. Prevent venous stasis


b. Promote maximum support
c. Prevent flexion deformities of the joints
d. Decrease edema around the joints

64. Nurse Ray writes a nursing diagnosis of activity intolerance related to lack of energy
conservation. Which of the following activities would Nurse Ray likely to choose to
implement in response to the diagnosis?

a. Encourage the patient to perform all tasks planned for the day.
b. Instruct the patient not to perform daily hygienic care until activity intolerance
improves.
c. Administer narcotics to promote pain relief and rest
d. Encourage the patient to alternate periods of rest and activity throughout the day.

65. The patient is ready for discharge. He tell Nurse Ray, “I know it is important to
exercise my joints, so I won’t lose mobility. But my joints are so stiff and painful that
exercise is difficult. “Which of the following responses by Nurse Ray is MOST
appropriate?
a. “Talk to your physician and tell him how you feel. Maybe he can increase the dosage
of your pain medication”.
b. “Take a warm bath before exercising. This may relieve some of your discomfort.”
c. “You are probably exercising too much. Decrease your exercise to every other day”.
d. “Stiffness and pain are part of your ailment. You can learn to cope by focusing on
activities you enjoy”.

Situation 14 - The nurse cares for several clients with acid- based imbalances.
66. The nurse is aware that a client who has experienced cardiac arrest is MOST at risk
for which of the following imbalances?

a. Metabolic Alkalosis
b. Respiratory Alkalosis
c. Metabolic Acidosis
d. Respiratory Acidosis

67. The nurse assists in the care of a 30 year old male post surgical client undergoing
nasogastric suctioning. The nurse understands that clients with nasogastrics suction is
MOST at risk with what imbalances?

a. Respiratory Acidosis
b. Metabolic Alkalosis
c. Metabolic Acidosis
d. Respiratory Acidosis

68. The nurse assesses a 32 year old female client who appears very anxious, restless
and irritable. The client has marked increase rate and depth of respirations. Based on
the information gathered, the client is experiencing which of the following imbalances?

a. Respiratory Alkalosis
b. Metabolic Acidosis
c. Metabolic Alkalosis
d. Respiratory Acidosis

69. The female client who is very anxious and fidgety is blowing off to much carbon
dioxide develops tingling sensation of the lips and fingers and is not able to control her
respirations. The most appropriate nursing intervention for the client is to ___:

a. Instrust the client to blow her nose and take deep breaths
b. Administer oxygen
c. Have the client breathe into a paper bag
d. Administer intravenous fluids as ordered

70. The nurse obtains a sample of a client’s arterial blood gas (ABGs). Which of the
following statements is NOT true about ABGs?

a. Interpretation of the client’s ABGs involves evaluation of pH, PCO2 and HCO3;
components of the ABGs.
b. ABGs assess the client’s oxygenation status and acid- base status.
c. ABGs provide information on blood parameters.
d. ABGs assess the client’s electrolyte and fluid balance.

Situation 15 - Mila, a 35 year old female was brought in the Emergency Unit complaining
of cold, clammy skin, disorientation, restlessness and confusion. The nurse suspects the
patient is developing cardiogenic shock.

71. Nurse Victor who admitted patient mila is aware that a decrease in cardiac output
results in a decrease in cerebral blood flow. Which of the following symptoms is one of
the earliest sign of cardiogenic shock?
a. Tachycardia
b. Altered level of consciousness
c. Presence of fourth heart sound (S4)
d. Decreased urine output

72. Nurse Victor assesses patient Mila. Decrease of which of the following factors would
help detect that the patient is at risk in developing cardiogenic shock? *

a. Heart Rate
b. Cerebral blood flow
c. Cardiac Index
d. Blood Pressure

73. Nurse Victor reviews the medical history of patient Mila. Which of the following
conditions is the MOST common cause of cardiogenic shock?

a. Decreased hemoglobin level


b. Hypotension
c. Acute myocardial infarction (MI)
d. Coronary Artery Disease

74. Which of the following procedure would be most helpful to nurse victor to confirm
the underlying cause of cardiogenic shock?

a. Monitoring pulmonary artery pressure


b. Monitoring cardiac enzymes
c. Monitoring mean arterial pressure using an intra-arterial line.
d. Monitoring central venous pressure

75. The physician prescribes a treatment plan for patient Mila. Nurse victor understands
that the initial treatment goal is which of the following?

a. Prevent infection
b. Correct metabolic acidosis
c. Correct hypoxia
d. Increase myocardial oxygen supply
Situation 16 – Ms. Dong-a, 35 years old, and a mother of two sought consultation at the
OPD. She claims that she felt a lump on her breast while doing a self-breast
examination. On further examination, Nurse Atilde confirmed that the lump has felt in
the upper outer quadrant of the right breast.

76. The nurse evaluates nurse Dong-a for breast cancer. Which of the following is a risk
factor for developing breast cancer?

A. Using foam contraceptives


B. Having an early menarche before the age of 12
C. Having an early menopause before the age of 45
D. Giving birth to a first child before the age of 20
77. The nurse recalls that there are other known risk factors for breast cancer such as
family history, environmental and dietary factors. Which of the following is the BEST
established dietary risk factor for breast cancer?

A. Fat-rich foods
B. Alcohol intake
C. Carbonated drink consumption
D. Caffeine consumption

78. The nurse is aware that for breast cancer patients, the single and MOST imporatant
predictor of outcome is the?

A. Presence of mastitis and an infection process upon diagnosis


B. Presence of nipple discharge at the time of diagnosis
C. Histological status of the axillary nodes
D. Age of the patient

79. Ms. Dong-a appeared to be worried and anxious. She ask the nurse what the doctor
meant when told that has Stage II cancer. The nurse explains that staging is based on
the size of the primary tumor. In stage II, the tumor is?

A. 3 cm in diameter and confined to the breast


B. Larger than 5 cm with involvement of the internal mammary lymph nodes
C. 2 cm or less in diameter and confined to the breast
D. Up to 5 cm or early metastasis to axillary lymph nodes

80. Based on further assessment, biopsy, results. Ms. Dong-a may have to undergo
mastectomy. The nurse recognizes that mastectomy is the treatment of choice when
which of the following conditions apply?

1. Tumor involves the nipple-areola complex


2. Tumor is larger than 7 cm
3. Tumor exhibits extensive intraductal disease involving multiple quadrants of the
breast
4. Patient cannot comply with daily radiation therapy

A. 1, 2, 3
B. 2, 3, 4
C. 1, 3, 4
D. 1, 2, 3, 4

Situation 17 – Nurse Carol participates in a study the purpose of which is to determine


the extent to which gender, age weight and height predict selected physiologic
outcomes, namely: forced expiratory volume in one second, hemoglobin concentration,
food intake and cancer-related weight change.

81. The independent variables of this study are the following, EXCEPT:

A. Height and weight


B. Age
C. Gender
D. Educational attainment

82. The dependent variables of this study are which of the following:

A. Cancer
B. Physical characteristics
C. Amount of serum concentration
D. Physiologic outcomes

83. Nurse Carol formulates the problem statement for the study. She understand that a
good problem has which of the following characteristics: The problem?

1. Clearly and unambiguously identifies the variables under consideration


2. Clearly expresses the relationship of the variables to one another
3. Specifies the nature of the population being studied
4. Implies the possibility of empirical testing

A. 1, 3 and 4
B. 1 and 3
C. All of the options
D. 2 and 4

84. Nurse Carol determines the participants in the study. The sample of the study will be
every 5th person in the oncology unit. A table of random numbers will be used to select
the beginning of the sample with in the first sampling interval. This is an example of
which sampling method?

A. Simple random sampling


B. Quota sampling
C. Stratified random sampling
D. Systematic sampling

85. Nurse Carol plans to use a method of data collection that use technical instruments
to collect data about the client’s physical, chemical, microbiological and anatomical
status. Which of the following is a method appropriate for the study?

A. Psychological
B. Physiological
C. Instrumentation
D. Pharmacological

Situation 18 – You and your friends are enjoying a quiet meal in a restaurant. Suddenly
you noticed that an adult male customer was holding his throat and apparently choking
on his food.

86. You approach the person who is apparently choking. What should you do FIRST?

A. Sweep the person’s mouth with your finger


B. Apply sharp upward thrusts over the person’s sternum
C. Determine if the person can make any verbal sounds
D. Hit the middle of the person’s back firmly
87. You prepare to perform the Heimlich maneuver on the conscious adult male. What
are the steps to be followed? Arrange the procedure in sequence?

1. Place the thump side of the first just above the umbilicus and well below and xiphoid
process
2. Make a fist
3. Stand behind the victim
4. Place arms around the victim’s waist
5. Perform five quick in and up thrusts (between the umbilicus and xiphoid process)

A. 2, 5, 3, 4, 1
B. 1, 2, 3, 4, 5
C. 4, 3, 1, 2, 5
D. 3, 4, 2, 1, 5

88. If the victim is unconscious, the sequence is:

1. Place the heel of one hand on top of the other between the umbilicus and xiphoid
process
2. Straddle the victim’s thighs
3. Give five thrusts in and up with the heel of the bottom hand
4. Make a fist

A. 1, 2, 3
B. 1, 2, 3, 4
C. 2, 1, 3
D. 2, 4, 1, 3

89. When you perform abdominal thrusts during the Heimlich maneuver, you are trying
to

A. Produce a burp to expel the foreign body


B. Pump the heart to push air out of the lungs
C. Produce a rush of air that expels the foreign body
D. Put pressure on the stomach to expel the foreign body

90. If the victim is pregnant or obese, which modification of the Heimlich maneuver
would be appropriate for this person?

A. Thrusts against the middle of the sternum rather than between the umbilicus and
xiphoid process
B. Perform thrust gently rather than forcefully and discontinue the thrusts after 6 tries, if
unsuccessful
C. Place the fist with the pinkle finger, rather than the thumb against the person’s body
D. Perform the thrusts with the person in the supine, rather than standing position

Situation 19 – The nurse cares for a 40 year old female admitted with a diagnosis of
chronic ulcerative colitis. The nurse tries to communicate therapeutically with the client.
91. The nurse recalls that this client is her third admission in the last 8 months. The
client is familiar with the nurse. Which of the following remarks by the nurse would be
MOST beneficial to the client?

A. “You are back again. What is wrong this time?”


B. “I don’t have to explain thigs to you. You have been here before anyway.”
C. “It’s been 3 months since you were last here. How do you feel about being back in
the hospital?”
D. “It’s nice to see you again. Do you miss the hospital?”

92. The physician assesses the client and recommends a partial bowel resection and an
ileostomy. The client tells the nurse, “My doctor likes to play smart. I’m sure the more
he operates, the better he likes it.” Which of the following responses by the nurse would
be MOST appropriate:

A. “Are you not being hard on your doctor? He is trying to help you.”
B. “What do you need by that?”
C. “You sound upset. We can talk about it, if you would like to.”
D. “Is your remark in any way have something to do with your operation?”

93. The client become increasingly irritable thinking about her operation. She is
unpleasant to her visitors and refuses her medication and treatments. Which of the
following nursing interventions would be therapeutic?

A. Encourage the client to direct her anger to her family members


B. Continue performing assigned tasks and duties as though nothing has happened
C. Encourage the client to discuss her feelings and concerns
D. Offer the client positive reinforcement everytime she cooperates with her medications
and treatments

94. The client refuses to eat her lunch and angrily tells the nurse to get out of her room.
What would be the nurse’s best response?

A. “I’ll get you something to relax you.”


B. “You sound angry. What is upsetting you?”
C. “Your anger does not bother me. I’ll back later.”
D. “I’ll leave, but you need to eat.”

95. The nurse makes arrangement for a former client who underwent ileostomy to talk
with the client. The reason for which is to:

A. Encourage the client and provide realistic information on ileostomy


B. Let the client know that she is not the only one who experienced the operation
C. Provide support for the plan of the physician regarding therapy of the client
D. Convince the client that she will not be disfigured and can still lead quality life

Situation 20 – Ms. Ada is a nurse working in the surgical unit. She is aware her legal
responsibilities as she assists in the care of post-operative patients.

96. Which principle is applicable in a situation where a sponge was left inside the
abdomen of a patient who had an exploratory laparotomy?
A. Doctrine of Force Majeure
B. Doctrine of Res Ipsa Loquitor
C. Doctrine of Viz major
D. Doctrine of Respondeat Superior

97. Under the Doctrine of Respondeat Superior, who among the following is liable if the
patient who had exploratory previously was reopened and a piece of gauze was found in
the abdominal cavity? The:

A. Instrument nurse
B. Surgeon
C. Operating room nurse
D. Assistant surgeon

98. The Doctrine of Respondeat Superior holds that:

A. The employer is responsible for the actions of his/her employee


B. The employer is not responsible for the action of his/her employee
C. The employee is not responsible for his/her own action
D. Using restraints without the patient’s permission constitue

99. Performing an act which a reasonable and prudent nurse would not do or the failure
to perform an act which a reasonable and prudent nurse would have done under similar
situation is construed as:

A. Misdemeanor
B. Malfeasance
C. Malpractice
D. Negligence

100. Ms. Ada learns of a nurse in the orthopedic ward who applied hot water bottle over
a paralyzed leg which consequently burned. This is an example:

A. Malfeasance
B. Misdemeanor
C. Malpractice
D. Negligence

NURSING PRACTICE V – CARE OF CLIENTS (PART C)


November 18, 2018

Situation 1- You’re assigned as a staff nurse in the Stroke Unit of the Medical Ward. One
of your responsibilities is to provide health teachings to the stroke patients and to the
other members of the family. The following questions refer to these statements.
1. You are teaching a patient about taking prophylactic warfarin sodium (Coumadin).
Which of these statement indicate that the patient understands how to take the drug.
SELECT all that apply:

1. The drug’s action peak in two hours


2. Maximum dosage is not achieved until 3 to 4 days after starting the medication
3. Effects of the drug continue 4 to 5 days after discontinuing the medication
4. Protamine Sulfate is the antidote for Warfarin
5. I should have my blood levels tested periodically

a. 2, 4, 5
b. 2, 3, 5
c. 1, 4, 5
d. 1, 3, 4

2. Maintaining oral hygiene for a stroke patient is important in his care. You are teaching
a family member how to give oral care to the stroke patient. Which of the following
nursing measure is NOT APPROPRIATE when giving oral care to such patient?

a. Cleaning the patient’s mouth and teeth with a soft-bristled toothbrush


b. Keeping portable suctioning equipment at the bedside
c. Opening the patient’s mouth with a padded tongue depressor
d. Placing the patient on his back with a small pillow under the head

3. Your patient has paraplegia. In giving instruction to the watcher when changing the
patient’s position in bed, which of the following is NOT APPROPRIATE?

a. Lifting the patient when moving him up in bed


b. Rolling the patient unto the side
c. Using a trapeze to help the patient lift off the bed
d. Sliding the patient to move up in bed

4. A patient is experiencing mood swings after stroke. Often times she has episodes of
crying that are distressed to the family members. What is the BEST TECHNIQUE that
you will advise the family members when the patient is having crying spells?

a. Ignore the patient during this time


b. Sit quietly with the patient until the crying episode is over
c. Tell the patient that the behavior is unacceptable
d. Attempt to divert the patient’s attention

5. To prevent joint deformities of the arm and hands of a hemiplegic patients what are
the APPROPRIATE position will you include in your health teachings of the family
member? SELECT all that apply.
a. 2, 3, 4
b. 1, 3, 5
c. 1, 2, 4
d. 2, 4, 5

Situation 2 – You are a community health nurse assisting a family in the care of their
daughter suffering from the past traumatic stress disorder
6. While caring for this client, the family notices that loud noises cause a serious anxiety
response. Which of the following explanations by you would help the family understand
the client’s response?

a. After a trauma, the client cannot respond to stimuli in an appropriate manner


b. Clients often experience extreme fear about normal environmental stimuli
c. Environmental triggers can cause the client to react emotionally
d. The response indicates that another emotion problem needs investigation

7. Which of the following instructions should you include about relationships for this
client with post-traumatic stress disorder? *

a. Assess the client’s discomfort when talking about feelings to family member.
b. Explain that avoiding emotional attachment protects against anxiety.
c. Warn the client that she will have a tendency to be over dependent in relationships
d. Encourage the client to resume former roles as soon as possible

8. Which of the following nursing intervention would best help this client and her family
handle interpersonal conflict at home? Have the family:

a. Discuss how to change dysfunctional family patterns


b. Teach the client to identify defensive behavior.
c. Agree not to tell the client what to do about problem
d. Arrange for the client to participate in social activities.

9. Which of the following nursing actions would be more appropriate when speaking with
this client about the trauma she experienced?

a. Listen attentively
b. Request the client write what is being said
c. Ask question to convey an interest in the details
d. Obtain validation of what the client says from another party

10. Which of the following nursing actions would you include in the plan of care for this
client who keeps saying that the trauma she experienced was just a case of bad luck?

A. Assist the client in defining the experience as a trauma


B. Encourage the client to verbalize the experience
C. Work with the client to accept positive and negative feelings
D. Help the client accept positive and negative feelings

Situation 3 – One of the competencies is expected of you as a beginning professional


nurse is to have a positive attitude toward research
11. You plan to study the effectiveness of films in teaching active exercises to post
stroke patients. Which of the following research designs will give you the most
confidence to answer the research problem?

a. time- series design


b. non-equivalent control group design
c. one-group pre-test, post-test design
d. post-test only, control group design

12. Which of the following sampling techniques is the most appropriate to answer a
research question on the effectiveness of active exercise on the mobility of post stroke
patients?

a. convenience sampling
b. purposive sampling
c. quota sampling
d. random sampling

13. You want to develop a tool for measuring the level of consciousness of stroke
patients. This is what type of research?

a. Historical study
b. Case study
c. Comparative study
d. Methodological study

14. You want to examine the difference in the eating behaviors between two groups of
high school students. Which of the following research designs would you use to answer
the research question?

a. Case study
b. Correlation study
c. Comparative design
d. Survey study

15. In a study on the relationship between personality make-up and obesity, a Pearson
r=.8 would mean:

a. a high, negative correlation


b. a moderate, positive correlation
c. a high, positive correlation
d. a low, positive correlation

Situation 4 – Ella, a 10-year old grade four pupil in a private school has just lost her best
friend, Lory, who died of Leukemia. Her mother brought her to the Emergency Room
who reported that Ella has been gloomy and often expresses feeling of joining Lory in
heaven.

16. Which of the following is the precipitating factor for Ella’s feeling of wanting to die?

a. The death of Lory


b. Her age
c. Her authoritative mother
d. Her fear of death

17. Ella’s feeling of joining Lory in heaven is manifestation of:


a. Suicidal intent
b. Suicidal threat
c. Paranoia
d. Suicidal ideation

18. Ella state “I will hang myself”. This is a manifestation of:

a. Denial
b. Suicidal threat
c. Suicidal intent
d. Paranoia

19. Which of the following is the best predictor of adolescents attempting suicide?

a. Depressed mood
b. Joyful mood
c. Feeling of euphoria
d. feeling of hopelessness

20. Ella state “I wish I were dead. I cannot stand anymore not having Lory around.” Your
most appropriate nursing action would be:

a. Do nothing because Ella will not do it


b. Refer Ella to your supervisor
c. Stay with Ella
d. Explore Ella’s feeling

Situation 5 – You are a staff nurse in the Out-Patient Department. Mrs. D brought her
9-year old daughter Bianca for consultation, Bianca is complaining of sore throat, muscle
tenderness, weakness of the arms and a general feeling of not being well. The attending
physician’s impression is infectious Polyneuritis.

21. Initially, what assessment data would be the BEST IMPORTANT for you to make?

a. Exposure to an infectious disease


b. Difficulty in swallowing
c. Difficulty in urinating
d. Diet intake for the last 24 hours

22. Bianca was admitted to the Pediatric ward. What would be your PRIORITY nursing
action when caring for her?

a. Providing a diversional activity for Bianca


b. Evaluating Bianca’s bilateral muscle strength
c. Making a game of range-of motion exercise
d. Assessing Bianca’s ability to follow simple commands

23. The pediatric ward nurse ask Bianca to cough every now and then. She also
assesses Bianca’s speech for decreased volume and clarity, the underlying rationale for
these assessments is to determine the ________.

a. regression to an earlier developmental phase


b. involvement of facial and cranial nerves
c. increased intracranial pressure
d. inflammation of the larynx and epiglottis

24. Assessment findings revealed that Bianca has weak gag and cough reflexes. During
this acute phase of Bianca’s illness, which of the following problems should receive the
HIGHEST PRIORITY?

a. Impaired swallowing related to neuromuscular impairment.


b. Risk for infection due to an altered immune system
c. Ineffective breathing pattern to neuromuscular impairment
d. Total urinary incontinence related to fluid losses

25. Bianca was placed on a medical ventilator. Which of the following nursing actions
should the ward nurse give emphasis to?

a. Turning Bianca slowly and gently from side to side to prevent respiratory complication
b. Engaging Bianca in vigorous passive range-of-motion
c. Maintaining Bianca in a supine position to prevent unnecessary nerve stimulation
d. Transferring Bianca to a bedside chair three times a day to prevent posture
hypotension

Situation 6 – Jennylyn Abad, 40 years old woman, has a history of Rheumatoid Arthritis
for the last ten years. The following refer to this situation.

26. Which of the following is common sign/symptom of rheumatoid arthritis?

a. Presence of crepitus over joints


b. Symptoms aggravated by humidity
c. Deformity and displacement of proximal joints
d. Presence of Heberden’s nodes

27. Which of the following would NOT be a common laboratory findings in rheumatoid
arthritis?

a. Low hemoglobin
b. Positive RA factor
c. Positive Lupus Erythematosus (LE) prep
d. Increase white blood count

28. Jennylyn has been taking steroids for the last five years to control her arthritis.
Which of the following is a common side effects of steroid therapy?

a. Hyponatremia
b. Hyperkalemia
c. Increase blood glucose
d. Protein anabolism

29. Which of the following diet is best suited to decrease the side effects of Jennylyn’s
steroid therapy?

a. Normal carbohydrate, high protein, high potassium


b. Low carbohydrate, high protein, high potassium
c. Low carbohydrate, high protein, low potassium
d. High carbohydrate, low protein, low potassium

30. Jennylyn was on Ibuprofen which has been stopped when the steroid therapy was
started. Which was the main reason for stopping the Ibuprofen once the steroid therapy
was started?

a. The two drugs taken together increase the risk of GI bleeding


b. The Ibuprofen would interfere with the action of the steroid
c. The Ibuprofen was not needed with the steroid
d. The two drugs together would cause severe liver damage

Situation 7 – You are a staff nurse in the Musculo-skeletal Unit of the hospital. You are
taking care of the patients with varied Musculo-skeletal conditions.

31. Which of the following inflammatory conditions commonly occurs in the shoulder?

1. Bursitis
2. Carpal Tunnel Syndrome
3. Tendonitis
4. Dupuy trens

a. 1 and 2
b. 2 and 3
c. 3 and 4
d. 1 and 3

32. Which of the following is NOT considered a conservative treatment for inflammatory
conditions of the shoulder?

a. Non-steroid Anti-inflammatory Drugs


b. Intermittent ice and heat application to the joint
c. Laser photo-therapy
d. Resting of the extremity

33. Impaired movement of the rotator cuff of the shoulder is generally termed as
Impingement Syndrome. Edema, which is an early manifestation of this syndrome, is
due to ________.

a. acute trauma of the shoulder


b. muscle spasm
c. hemorrhage of the structures involved
d. atrophy due to disuse

34. For a persistent shoulder pain and weakness, this usual treatment modality
performed is _______.

a. Pulse-magnetic field
b. Laser photo-therapy
c. Radio-frequency ablation
d. Arthroscopic Synovectomy

35. One of the most common foot problems is Callus. This condition precedes the
formation of a Callus _____.

a. External pressure on the foot


b. Faulty foot mechanics
c. Flexion deformity of the interphalangeal joint
d. Penetrating nail plate in the surrounding skin of the foot

Situation 8 – You are a staff nurse working in a hospital and encounter situations with
ethico-moral implications.

36. Which of the following nursing actions will you do if the patient is unconscious and
unable to make decisions? This fact should be:

a. reported to the nurse supervisor


b. reported to the attending physician
c. documented in the patient’s chart
d. reported to the police

37. A patient came in from prison for emergency treatment. Which of the following
precautionary measures will you institute to ensure the safety of the hospital staff and
the other patients?

a. Provide the same assessment and care to the client


b. Use restraints
c. Lock the ER door against entry if security is in question
d. Never release the hand or ankle restraint

38. In using restraints to patients, which of the following guidelines will you follow?

a. Apply restraints because the patient needs it


b. Apply restraints according to the hospital policy
c. Restraint is the most effective nursing intervention to minimize aggressive behavior
d. Maintain respect for the client when applying restraints

39. You have observed that a group of students are implementing their research project
on the effect of hot and cold application to febrile patients without informing the
participants properly. What ethical principle was violated by the students?

a. Nonmaleficence
b. Autonomy
c. Justice
d. Respect for person

40. A researcher should pay particular attention in protecting the rights of certain
vulnerable groups. Which of the following is NOT considered vulnerable?

a. Prisoners
b. Children
c. Mentally-challenged persons
d. Postpartum mother with normal delivery

Situation 9 – You are a staff nurse in the Neurologic Unit of the Pediatric Department.
Several problems have been reported in relation to ventriculo-peritoneal shunt for
hydrocephalus patients. The initial investigation on the care of patient with this shunt
revealed that majority of the staff nurses have little experience on caring for such
patients. It was decided to look into their level of knowledge and skills to serve a
baseline data for improving the nursing care of hydrocephalic patients on
ventricular-peritoneal shunt. The following questions refer to this situation.

41. A four-year old hydrocephalic preschooler is scheduled to have a


ventriculo-peritoneal shunt at the right side of the head. Which of the following positions
should the child be placed immediately after surgery?

a. Supine, with the head of the bed flat


b. On the left side with the head of the flat
c. Prone, with the head of the bed elevated
d. On the right side, with the foot of the bed elevated

42. What nursing action should the nurse do when providing post-operative nursing care
to a child after insertion of a ventriculo-peritoneal shunt?

a. Monitoring for the increased temperature


b. Check the urine for glucose and protein
c. Administer narcotics for pain control
d. Test cerebrospinal fluid leakage for protein

43. An infant who has hydrocephalus is irritable, lethargic, and difficult to feed before
the ventriculo-peritoneal shunt is placed. To maintain the infant’s nutritional status,
which of the following nursing action is the MOST APPROPRIATE?

a. Feeding the infant just before doing any procedures


b. Giving the infant small, frequent feedings
c. Scheduling the feeding every six hours
d. Feeding the infant in a horizontal position

44. The parent of a school-age child with ventriculo-peritoneal shunt were given
discharge teaching. Which of the following signs of a blocked shunt would the parents be
able to identify that the discharge teaching is successful?

a. Irritability and increase difficulty with eating


b. Elevated temperature and reddened incisional site
c. Decreased urine output with stable intake
d. Tensed fontanel and increased head circumference

45. After a six-year old child underwent placement of ventriculo-peritoneal shunt, he


was put on I.V. medication. The head nurse observed that many of the staff nurses are
disconnecting the flush syringe first and then clamping the intermittent infusion device.
The staff nurse seem not to understand the benefits of positive pressure techniques.
Which of the following strategies would be the MOST EFFECTIVE way to improve the
staff nurses technique in giving I.V. medication?

a. Post an evidence-based article on the topic in the unit


b. Create a poster presentation on the topic with a required post test
c. Send a memorandum on the importance of clamping the device first
d. Ask each nurse if they are aware that their practice is not current

Situation 10 – You are a staff nurse in the Stroke Unit of the Medical Ward. You have a
stroke patient with a tracheostomy. In the Unit’s last meeting it was discussed that there
has been increasing complaints on the nurses incompetence in caring for a patient with
tracheostomy. The following questions relate to this situation.

46. In providing tracheostomy care which of the following is the nurse’s PRIORITY
nursing action? The nurse _____.

a. cut the dressing using sterile scissors


b. cleans the incision with iodine-based antiseptic
c. secure clean ties before removing soiled ones
d. used clean technique

47. Which of the following nursing actions should the nurse TEACH the patient’s care
giver regarding tracheostomy care?

a. Remove the tracheostomy tube if obstruction occurs


b. Leave the tracheostomy ties in place until the physician change them
c. Remove and clean the inner cannula daily
d. Never put a covering over the tracheostomy stoma

48. After a tracheostomy procedure, the nurse documents important observations made
during the procedures such as _______.

a. response of the patient after the procedure


b. vital signs of the patient during the procedure
c. amount, color, and consistency of sputum and appearance of the incision
d. behavior of the patient during the procedure

49. What is the purpose of a tracheostomy? To _____.

a. establish an airway
b. establish a pathway for nutrition
c. monitor respiratory functions
d. maintain the patency of the airway

50. Which of the following can be a MAJOR PROBLEM for a patient with tracheostomy?

a. Breathing
b. Ambulating
c. Swallowing
d. Singing
Situation 11- You are staff nurse in the Psychiatric Unit of a private hospital. Lorena, a
20-year old stage actress was admitted with the chief complaints of getting angry easily
and inability to tolerate being alone. She claimed that she has also the tendency to
manipulate people and feels unhappy most of the time. She was diagnosed to be
suffering from Borderline Personality Disorder (BPD).

51. You have observed that Lorena is manifesting “spitting”. This characteristic of BPD is
BEST defined as ______.

a. viewing people and objects as parts, either good or bad


b. having two personalities
c. talking about other people behind their back
d. literally spitting in other people’s face

52. When assessing a patient with BPD which of the following information would you
focus on?

a. Ability to get people on his/her side


b. Disruption in some aspect of his/her life
c. desire for intimate relationship
d. Increase acceptance from other people

53. Lorena revealed she has a tendency to manipulate others. What would be the MOST
APPROPRIATE short-term goal for her? For Lorena to _____.

a. have an intimate relationship


b. acknowledge her own behavior
c, stop arguing with other people
d. express her feelings verbally

54. Patient with BPD manifest transient psychotic symptoms. What is the drug of choice
to treat these symptoms?

a. Mood stabilizer
b. Benzodiazepines
c. Lithium
d. Antipsychotics

55. Lorena has a history of alcohol abuse so she was started on Antabuse. Which of the
following is a COMMON side effect of this drug?

a. Hypertension
b. Bradycardia
c. Depression
d. Elation
Situation 12- Angela, a 40-year old, single mom of two preschool children, aged two (2)
and five (5) years old respectively was admitted to the Psychiatric Unit for attempting to
kill her two children.

56. Angela says “My dead mother ordered me to kill my two children before the devil
could get them”. What behavior is being manifested by Angela?
a. Auditory hallucination
b. Visual hallucination
c. Tactile stimulation
d. Olfactory hallucination

57. You ask Angela to touch a 25-centavo coin and a one-peso coin. You have observed
that Angels was having difficulty differentiating one coin from the other. This is
manifestation of _____.

a. astereognosis
b. kinesthetic
c. agraphestesia
d. cenesthetic

58. You have observed that Angela is manifesting persistent hallucination. What is the
MOST EFFECTIVE therapy in treating patient with a condition like that of Angela’s? *

a. Cognitive – behavior therapy


b. Crisis intervention
c. Psycho- pharmacologic treatment
d. Anti – psychotic drug

A. c and d
B. a and b
C. b and d
D. a, b, and c

59. Which of the following is the GOAL of nursing intervention for a patient who is
hallucination.

a. To help the patient to have an increased awareness that in symptoms she is


experiencing is not real
b. To help the patient to identify what triggers her hallucination
c. To encourage the patient to verbalized her fear, anxiety, and anger
d. To help the patient to go back to her real world

60. What communication technique would be Most effective to achieve the nursing goal
for patient with hallucination?

a. Listening
b. Facilitative communication
c. One-on-one discussion
d. Social interaction
Situation 13- Charting is part of the professional nurse’s responsibility related to record
management. The following questions refer to this statement.

61. One of the characteristics of charting is brevity. Which of the following example of
charting shows this characteristics?

a. Nurse brought the patient to OR via stretcher at 10:15


b. Patient left for surgery via stretcher at 10:15 am
c. To surgery via stretcher at 10:15
d. patient brought by the nurse to OR via stretcher at 10:15 accompanied by the
“bantay”

62. Which of the following is NOT a requirement for a late entry in charting?

a. nurse’s initials above the time


b. may be made if something has been forgotten
c. circle it and write “late entry”
d. reason for the late entry

63. There is a blank space after your last entry in your charting. You will ______.

a. draw a double line after your last entry


b. sign your full name after your last entry
c. draw a horizontal line through the center of an empty line
d. draw a perpendicular line across the empty space

64. Which of the following should the nurse’s notes focus on?

a. Immediate past and the present


b. The present only
c. The future
d. The recent past

65. Which of the following sample charting would show the characteristic of ACCURACY?

a. intake from 700-1000 ml: 80 ml of coffee; 240 ml of orange juice; 500ml of water
b. Patient on forced fluid but refused to take it most of the time
c. Patient on forced fluids observed
d. Given fluid at frequent intervals but takes only a few sips

Situation 14- During summer season incidence of heat stroke rises. As a community
health nurse you should disseminate information on this as part of your role as a health
educator.

66. A patient came in with sign of heat stroke. Which of the following are
signs/symptoms of profound CNS dysfunction?

a. Elevated temperature
b. Hot, dry skin
c. Tachypnea, hypotension, tachycardia
d. Confusion, delirium, bizarre behavior
67. Which of the following is the primary goal in the care of patient with a heat stroke?

a. To maintain cardiac functions


b. To reduce immediately the high temperature
c. To restore normal body temperature
d. To prevent further complication
68. Which of the following is the best advice you would give to an athlete to prevent
heat stroke?

a. To monitor fluid losses and weight lost during workout activities and to replace fluids.
b. To avoid immediate exposure to high temperature
c. To maintain adequate fluid intake
d. To avoid planning outdoor activities between 10 a.m. and 2 p.m.

69. To prevent heat stroke, the following pieces of advice are given to the community,
EXCEPT:

a. Avoid direct exposure to the sun especially late in the morning


b. Always drink plenty of water
c. Use a hat or an umbrella when going out of the house
d. Just stay home and relax

70. Which of the following would your best advice to the community people during
summer time to prevent dehydration?

a. Use a hat or an umbrella when going out of the house


b. Just stay home and relax
c. drink plenty of fluids
d. wear light-colored clothing

Situation 15- Lizbeth, a 30-year old registered nurse with two children, legally separated
from her husband was admitted to the psychiatric unit three weeks ago.

71. You are the nurse attending to Lizbeth. You have observed that she has a habit of
washing her hands repeatedly for a long period of time. This is a manifestation of what
kind of behavior?

a. Negative
b. Hyperactive
c. Ritualistic
d. Nonconformist

72. Lizbeth engages in this behavior to _____.

a. protect herself from undesirable people


b. relieve her anxiety
c. occupy herself with purposeful activity
d. call the attention of other
73. A new nurse introduce herself to Lizbeth and asks her name. Lizbeth responds “I am
an obsessive- compulsive neurotic. I have had psychoanalysis for ten years. What do
you think can you do for me?” Your BEST response would be _____.

a. “Can we talk about that Lizbeth?”


b.” I need to know you better Lizbeth”
c.” You seem to feel hopeless”
d.” who was your psychoanalyst?”
74. Lizbeth tells you “That the new nurse makes me angry. Like you, she does not
understand what my real problem is”. Your BEST reply would be _____.

a. “You seems to be upset. I will come back later”


b. “You have the right to be upset when people don’t seem to understand”
c. “That’s a common feeling. I understand. Let’s talk about it”
d. “I know what your problem is. You are an obsessive-compulsive personality”

75. Diazepam (Valium) was prescribe for Lizbeth. You gave her instructions on effects of
the drug. What statement would indicate that Lizbeth needs further health teaching
about the medication?

a. “I’m so glad I can still eat chocolate while I’m taking this.”
b. “I’m so glad no blood tests are necessary while I’m taking this.”
c. “I’m so glad Valiums won’t affect my driving skills”
d. “I’m so glad I will only have to take this until I learn to be less anxious”

Situation 16- As a professional nurse, one of your roles is an advocate for the patient’s
rights. The following questions refer to this statement.

76. You were invited by a women’s organization to be the resource speaker on “Violence
against women and children”. Women and their children are protected against violence
under Republic Act No.__.

a. 9173
b. 9211
c. 9262
d. 9160

77. One of the violence acts is sexual violence. Which of the following does NOT
constitute sexual violence?

a. Acts of lasciviousness
b. Rape
c. Treating a child as a sex object
d. Economic abuse

78. What would be the nurse’s MAIN concern when a case of abuse, maltreatment, or
neglect is suspected in a patient?

a. Reporting the incident to proper authorities


b. Safety and welfare of the patient
c. Referring the patient to a mental health worker
d. Just keep quiet about the matter

79. What is the MOST common clinical manifestation of child neglect?

a. Missed appointment with health care provider


b. Fracture
c. Unexplained bruises
d. Malnutrition and dehydration
80. What should be the FOCUS of care for patients who are suspected to be abused,
maltreated, or neglected?

a. Immediate treatment of any injury


b. Consequences of the act
c. Prevention of further injury
d. Referral to legal authorities

Situation 17- You are a staff nurse at the Ear Unit of the Medical Ward of a government
hospital. Clarita, a 25-year old stage actress was admitted with the chief complaint of on
and off tinnitus at the right ear. The audiogram confirms conductive hearing loss or
mixed loss especially in the low frequencies. Clarita was diagnosed with Otosclerosis.

81. Otosclerosis is a common cause of what condition?

a. Premature labor
b. Meningitis Auditory nerve
c. Sensori-neural hearing loss
d. Conductive hearing loss

82. The audiogram also confirms a sensori-neural hearing loss. What ear structure is
damaged when this condition is present?

a. Outer and/or middle ear


b. Tympanic membrane
c. Cochlear nerve
d. Stapes

83. Clarita was given Sodium Flouride. What is the rationale for giving this to her?

a. To mature the spongy bone growth


b. To remove the diseased stapes
c. To prevent further complications
d. To restore hearing

84. A surgical procedure was recommended for Clarita. Which of the following surgical
procedures is useful in correcting her condition?

a. Myringotomy
b. Ossicular reconstruction
c. Cochlear transplant
d. to restore hearing

85. After her surgery, Clarita was placed on her left side with the head of the bed
elevated. What is the rationale for placing her in this position?

a. To minimize the pressure in the middle ear


b. To prevent complication of bleeding
c. To ensure patient safety
d. To prevent vomiting
Situation 18- Ms. Ligaya Co is a chief nurse in a secondary level hospital with nursing
staff consisting of registered staff nurses, nursing assistants and aides.

86. Ms. Co stresses the importance of promoting ‘esprit d corps’ among the nursing
staff. Which of the following statement indicates that they understand the meaning of
the term?

a. “In order that we achieve the goals of the institute, we must follow the directives
coming from above.”
b. “We will ensure that all resources we need are available when needed”
c. “Let’s work together in harmony; we need to be supportive of one another.”
d. “We need to show our competence to the higher management level.”

87. She assert the importance of promoting a positive organizational culture among the
nursing staff. Which of the following behaviors indicate that this has been attained? *

a. Obedient and uncomplaining


b. Powerful and oppositional
c. Competitive and perfectionist
d. Caring and nurturing with one another

88. Ms. Co is a visionary strategist, and desires to be a committed leader. Which of the
following types of leadership do these behaviors reflect?

a. Servant leadership style of politicians


b. Transactional structure, order of the org
c. Connective connecting people
d. Transformational

89. She knows that as a leader, she has to strategize in order to create followership in
response to authority. This capacity to act or the strength to accomplish a goal is
referred to as _____.

a. power
b. accountability
c. responsibility
d. authority

90. Ms. Co gathered the nursing staff and other health care worker in the hospital who
support the idea of having a meeting to discuss problems that affect the delivery of care
and to agree to speak with one voice. This power mechanism is called ______.
a. coalition
b. upward appeal
c. rationality
d. assertiveness

Situation 19- Ana, a 28-year old supervisor in a business processing office, consult at
the Out-patient department of the hospital with the chief complaints of headache,
drowsiness, nausea, and vomiting. She revealed that one week before consult she
stopped drinking coffee which she used to take three to five times a day. She was
diagnosed with Caffeine Discontinuation Syndrome (CDS).

91. What are the effects of drinking coffee two to three cups a day on the central
nervous system?

a. Feeling motivated and energized, less fatigue


b. Feeling motivated and energized, headache
c. Feeling motivated and drowsy, headache
d. Headache, less fatigue

92. Which of the following reinforces a person to continue drinking coffee?

a. Feeling of unpleasant symptoms when drinking is abruptly stopped.


b. Drinking coffee at Starbucks is the “in-thing
c. Feeling of well-being
d. The price of coffee is getting lower

93. Ana was given Luminal 60 mg./day p.o. this drugs is a/an _____/

a. intermediate-acting sedative
b. short-acting sedative
c. hypnotic drug
d. long-acting sedative

94. While doing your assessment Ana complained of nausea. Which of the following
would be you PRIORITY nursing action?

a. Refer Ana to the attending physician immediately


b. Let Ana put her head in-between her legs
c. Administer an anti-emetic drug
d. Offering Ana some ice chips

95. You have observed Ana to be pacing in the room, restless, and stutters when your
speaking to her. You would assess Ana as manifesting which of the following global
anxiety response?

a. Cognitive
b. Behavioral
c. Motor
d. Biological

Situation 20- You are a staff nurse in a psychiatric unit, use of therapeutic
communication is one of your nursing responsibilities.

96. Verbal communication is the use of words when talking to your patient.

a. understandable phase
b. content
c. the circumstances
d. understandable sentences
97. Context of a verbal communication is the:

a. use of understandable sentences


b. use of literal words
c. use of clear sentence
d. environmental where communication occurs

98. Non-verbal communication is the behavior that accompanies verbal communication.


Which of the following is NOT an indicator of this?

a. Eye Contact
b. Word representing an object
c. Grunts and groans
d. Bochy language

99. Which of the following give meaning and context to the message?

a. Process
b. Phrases and sentences
c. Context
d. Thoughts and feelings

100. Which of the following situation is an example of incongruent message?

a. When the nurse means what she says


b. When the words and behavior of the nurse agree
c. When what the nurse says and does do not agree
d. When content and process agree

You might also like